Behavioral Health: Practice Q's

अब Quizwiz के साथ अपने होमवर्क और परीक्षाओं को एस करें!

A mother presents with her 8-year-old son for evaluation of his sudden misconduct at home and in school. She states that he has become defiant and has failed multiple assignments at school since the birth of his sister two months ago. The patient tells you he "feels sad" that he has to share his parents' attention with his new sibling. Which of the following is the best diagnosis? A. Adjustment disorder B. Conduct disorder C. Major depressive disorder D. Oppositional defiant disorder

A. Adjustment disorder Adjustment disorder -Behavioral response -Develops ≤ 3 months after onset of stressor -Reaction is excessive -Symptoms resolve by 6 months Psychotherapy is the first-line treatment strategy for managing adjustment disorder. Therapy focuses on enabling the patient to remove the stressor from their life, teaching the patient to adapt to stressors that cannot be removed, and altering the response to the stressor with symptom reduction and behavioral modification. Medication therapy may be used as adjunct management for those whose condition does not improve with therapy alone. Antidepressants (e.g., fluoxetine, sertraline, venlafaxine) can be used to combat depression, while benzodiazepines (e.g., lorazepam, diazepam) may alleviate symptoms of anxiety or insomnia.

A 24-year-old woman presents to the psychiatry clinic with treatment-resistant depression. Her psychiatrist decides to initiate a monoamine oxidase inhibitor. What food should this patient avoid eating while taking a monoamine oxidase inhibitor? A. Aged cheeses B. Cottage cheese C. Decaffeinated coffee D. Salted crackers

A. Aged cheese Tyramine Reaction MAOIs Tyramine containing foods (alcohol, cheeses) HTN Manage the blood pressure Supportive care

A 26-year-old, overweight woman presents to the clinic concerned about her body image. She states that for years she has frequently used diuretics and laxatives to lose weight. Which of the following is first-line pharmacologic therapy for the most likely diagnosis? A. Fluoxetine B. Lithium C. Olanzapine D. Orlistat

A. Fluoxetine Bulimia Nervosa Patient with a history of binge eating followed by purging via forced vomiting, purging via laxative misuse, driven exercise, or fasting PE will show dental erosions and callused knuckles Diagnosis requires at least one episode of binge eating with inappropriate compensatory behavior per week for a minimum of 3 months in a patient whose self-evaluation is unduly influenced by body shape or weight Treatment is cognitive behavioral therapy, SSRIs, or both

A 65-year-old man who does not have housing and has a history of diabetes mellitus and post-traumatic stress disorder is admitted to inpatient psychiatry approximately 24 hours after presenting to the emergency department with tremulousness and a report of "two voices laughing at me." Upon initial psychiatric assessment, the patient states, "the voices are terrifying...they sound like demons" and then clarifies "but they aren't bothering me anymore." His ethanol level on admission was 300 mg/dL. Nursing notes that the patient slept 3.25 hours and appeared agitated at 02:00. Today, his morning vital signs are temperature 99.0°F, HR 98, BP 165/88, and RR 20. Based on his presentation, what is the primary diagnosis for this patient? A. Alcohol hallucinosis B. Brief psychotic disorder C. Delirium tremens D. Schizophrenia

A. Alcohol hallucinosis Those who drink heavily may experience alcohol hallucinosis approximately 12-24 hours after the cessation of ethanol consumption. Unlike patients with primary psychotic disorders, such as schizophrenia, patients with alcohol hallucinosis tend to reconstitute more quickly and rarely suffer prolonged hallucination. Visual and tactile hallucinations are the most frequent form of hallucination during alcohol withdrawal, but schizophrenia-like auditory hallucinations may also occur. Antipsychotics may be given to reduce emergent agitation during alcohol hallucinosis but should be used sparingly. Antipsychotics may induce extrapyramidal symptoms or neuroleptic malignant syndrome and are not indicated for long-term use in patients with secondary psychoses. Instead, benzodiazepines, such as lorazepam or chlordiazepoxide, should be administered to end ethanol withdrawal during alcohol hallucinosis. Unlike methamphetamine-induced psychosis, which typically begins during intoxication, alcohol hallucinosis usually occurs during withdrawal.

A 33-year-old homeless woman comes to the emergency department because of hallucinations. Security guards found her unconscious at the parking lot of a mall. After paramedics arrived and stabilized her, she became very aggressive and started screaming at them to help her "get rid of the ants crawling all over [her]." Which of the following conditions is most likely to cause this type of behavior? A. Alcohol withdrawal B. Brain tumor C. Epilepsy D. Hypoacusis E. Schizophrenia

A. Alcohol withdrawal Many conditions can cause multiple types of hallucinations. For example, alcohol withdrawal syndrome can cause tactile, visual, or auditory hallucinations; however with this condition, visual and tactile are more common than auditory hallucinations.

Which of the following findings is a negative symptom of schizophrenia? A. Alogia B. Auditory hallucinations C. Delusions D. Disorganized speech

A. Alogia The symptoms of schizophrenia can be divided into positive and negative symptoms. Positive symptoms are marked by the presence of something that is usually not present, such as delusions, hallucinations, disorganized speech, and disorganized behavior. Negative symptoms are marked by the absence of something that is normally present and include flat affect, poverty of speech (very brief speech), alogia (inability to speak), anhedonia (lack of pleasure), poor attention, and social withdrawal.

Which of the following antidepressants is known to have the most severe toxicity in overdose? A. Amitriptyline B. Fluoxetine C. Sertraline D. Venlafaxine

A. Amitriptyline Tricyclic antidepressants, such as amitriptyline, are dangerous in cases of intentional or unintentional overdose. Tricyclic antidepressants were used extensively in the management of depression and other psychiatric disorders between the 1950s and 1980s. Although selective serotonin reuptake inhibitors have reduced the frequency of use of tricyclic antidepressants, tricyclic antidepressants are still frequently used and are dangerous in overdose. Tricyclic antidepressants primarily work by inhibiting the reuptake of serotonin and norepinephrine. However, adverse effects (especially in overdose) are due to blockage of cardiac sodium channels, antagonism of central and peripheral muscarinic acetylcholine receptors, antagonism of peripheral alpha-1 adrenergic receptors, antagonism of histamine (H1) receptors, and antagonism of central nervous system gamma-aminobutyric acid A receptors.

A 20-year-old college athlete presents with fatigue and cold intolerance. She exercises five hours per day because she thinks she is obese and wants to lose weight. Physical examination reveals a body mass index of 15 kg/m2, hypotension, bradycardia, and dry skin. Laboratory studies reveal low luteinizing hormones but are otherwise normal. Which of the following is the most likely diagnosis? A. Anorexia nervosa B. Bulimia nervosa C. Hyperthyroidism D. Schizophrenia

A. Anorexia nervosa Anorexia Nervosa Three core features Restriction of energy intake relative to requirements leads to significantly low body weight Intense fear of gaining weight or persistent behavior interfering with weight gain Disturbance in body image Two types Restricting Binge-eating and purging type Recommended interventions Determination of setting Medical stabilization Nutritional rehabilitation Psychotherapy For youth, best evidence is for family-based therapy, including parental management of ongoing weight restoration

Which of the following disorders belongs to the cluster B personality disorders? A. Antisocial personality disorder B. Dependent personality disorder C. Obsessive-compulsive personality disorder D. Paranoid personality disorder

A. Antisocial personality disorder Personality is defined as a pattern of perception, relation, and thought about one's environment and self that manifests across several different contexts. When an individual's personality traits are maladaptive and inflexible in a variety of situations, such that they cause impairment, distress, and significant deviation from the cultural norms, then a personality disorder may be diagnosed. Personality disorders tend to affect men more than women, as well as those who are younger, uneducated, and unemployed, although there are different epidemiologic differences between the different disorders. There are 10 different personality disorders that are divided into three clusters: A, B, and C. Cluster B is the dramatic and emotional cluster that includes antisocial, borderline, histrionic, and narcissistic personality disorders. Antisocial personality disorder is diagnosed starting at age 18 years using criteria from the Diagnostic and Statistical Manual, Fifth Edition. Patients with antisocial personality disorder present with a consistent pattern of disregarding and violating the rules of society and the rights of others.

A 25-year-old man presents to the emergency department after being arrested for stealing from a department store. He states that he is the president of the United States. He also says that he has not slept for three days and does not feel like he needs to sleep. During the interview, he speaks rapidly and is easily distracted. He denies any use of substances, and the toxicology screen is negative. Which of the following is the most likely diagnosis? A. Bipolar disorder type 1 B. Bipolar disorder type 2 C. Cyclothymic disorder D. Major depressive disorder

A. Bipolar disorder type 1 Bipolar disorder is a mood disorder that involves mania and hypomania as well as depression. The subtypes of bipolar disorder include bipolar type 1 and bipolar type 2. Patients with bipolar type 1 disorder experience manic episodes and nearly always experience major depressive and hypomanic episodes. Bipolar type 2 disorder is marked by at least one hypomanic episode, at least one major depressive episode, and the absence of manic episodes. The most important risk factor for bipolar disorder is a family history of bipolar disorder. The average age of onset is between 20 and 30 years of age, and onset after age 50 is rare. Bipolar disorder can initially present with mania, hypomania, major depression, or mixed features. Mania is defined by a distinct period of abnormally and persistently elevated, expansive, or irritable mood and abnormally and persistently increased activity or energy, lasting at least one week and present most of the day, nearly every day, or for any duration, if hospitalization is necessary. In addition, during this period of mood disturbance and increased energy or activity, three (or more) of the following symptoms must be present: inflated self-esteem or grandiosity, decreased need for sleep (feels rested after only three hours of sleep), more talkative than usual or pressure to keep talking, flight of ideas or subjective experience that thoughts are racing, distractibility (attention too easily drawn to unimportant or irrelevant external stimuli), increase in goal-directed activity (either socially, at work or school, or sexually) or psychomotor agitation (purposeless non-goal directed activity), and excessive involvement in activities that have a high potential for painful consequences, such as unrestrained buying sprees or sexual indiscretions. Patients with irritable, but not elevated or expansive mood, require four (rather than three) of the above symptoms to confirm the diagnosis. The presence of at least one manic episode is diagnostic of bipolar disorder type 1. However, the mood disturbance must be severe enough to cause significant impairment in function and must not be due to an illicit substance. Although manic episodes are required for the diagnosis of bipolar disorder type 1, most individuals with bipolar disorder type 1 spend far more time in depressive episodes than in mania. The management of bipolar disorder involves maintenance treatment (preventive treatment to prevent recurrence of mood episodes and allow higher psychosocial function) and treating acute mood episodes, including depression, hypomania, and manic episodes. The initial maintenance regimen is typically the same regimen that improved the initial acute mood episode. Patients who have recurrent mood episodes should be treated with lithium for maintenance therapy. Lithium is the only maintenance medication used in the treatment of bipolar disorder type 1 that is associated with reducing the risk of suicide. Other medications that can be used for maintenance therapy of bipolar disorder include valproate, lamotrigine, or quetiapine. Combination therapy with a mood-stabilizing agent (e.g., lithium, valproate, or lamotrigine) and an antipsychotic is used in individuals who do not have adequate control with monotherapy. Acute manic or hypomanic episodes may be treated with lithium, valproate, or antipsychotics. Lithium or valproate and an antipsychotic may be necessary in severe cases. Patients with bipolar disorder type 1 disorder and acute depressive episodes may be treated with valproate, lithium, lamotrigine, or quetiapine. Approximately 10-15% of individuals with bipolar disorder die by suicide, and preventing suicide should be one of the goals of management.

A 35-year-old businessman presents to the emergency department requesting to be seen first and by the most experienced physician. He states that he needs to be evaluated immediately because he must return to work right away as he is extremely important to his business. He informs all the healthcare staff he interacts with that he is a rich and famous businessman. Which of the following is an additional common feature of this personality disorder? A. Fragile self-esteem B. Perceptual and cognitive disturbances C. Preoccupation with minute details at the expense of efficiency D. Suspiciousness of others

A. Fragile self-esteem Narcissistic Personality Disorder Arrogant Entitlement Lack of empathy Self-importance Superiority Treatment: psychotherapy Addition of mood stabilizers or antipsychotics if patient poses threat to self or others

A 35-year-old woman presents with anhedonia and depressed mood for two weeks. In the past four days, she has been easily distractible and irritable, but her occupation is not affected. Which of the following is the most likely diagnosis? A. Bipolar disorder type II B. Cyclothymic disorder C. Dysthymic disorder D. Major depressive disorder

A. Bipolar disorder type II Hypomanic episodes include abnormally and persistently elevated, expansive, or irritable mood with at least three manic symptoms. Hypomania often does not present with psychotic symptoms, racing thoughts, or excess psychomotor agitation. Social and occupational functioning is not significantly affected, and psychotic features are absent. A mixed episode is manifested by rapidly alternating moods, with symptoms of both a manic episode and a depressive episode, lasting at least one week. Social and occupational impairment may be present. Bipolar disorder type II is more common in women than men. The DSM-5 diagnostic criteria for bipolar disorder type II requires the presence of at least one hypomanic episode and at least one major depressive episode and the absence of manic episodes. Mood stabilizers, such as lithium, carbamazepine, and valproic acid, are effective for treating bipolar disorder type II. Second-generation antipsychotics (olanzapine, aripiprazole, quetiapine, ziprasidone) can treat hypomanic episodes. For acute depressive episodes, selective serotonin reuptake inhibitors (fluoxetine, paroxetine, sertraline) or quetiapine are the drugs of choice. Olanzapine plus fluoxetine has shown to be helpful and may be more effective than the medications used individually. Family therapy, social therapy, group therapy, and psychotherapy should be added to the treatment regimen.

A patient with symptoms of major depression is seeking help to stop smoking. Which of the following medications would be most suitable to treat both conditions simultaneously? A. Bupropion B. Fluoxetine C. Nicotine patch D. Varenicline

A. Bupropion Tobacco Use/Cessation Motivational interviewing Nicotine replacement Bupropion (avoid with seizure or eating disorders) Varenicline Consider combination therapy USPSTF, Jan. 2021: insufficient evidence to recommend for or against pharmacotherapy for smoking cessation in pregnant persons

A 25-year-old man with refractory schizophrenia who is being treated with multiple medications has developed left-sided chest pain. Laboratory studies show decreased white blood cell count, decreased absolute neutrophil count, and elevated troponin. Which of the following medications is he taking? A. Clozapine B. Olanzapine C. Risperidone D. Ziprasidone

A. Clozapine Schizophrenia Two or more symptoms Delusions, hallucinations, disorganized speech, grossly disorganized or catatonic behavior, negative symptoms Brief psychotic disorder: < 1 month Schizophrenia: > 6 months Schizophreniform disorder: 1-6 months Schizoaffective disorder: psychosis + mania or depression

Which of the following clinical findings supports a diagnosis of schizotypal personality disorder? A. Cognitive and perceptual disturbances B. Depressed mood daily C. Frank psychosis D. Manic episodes

A. Cognitive and perceptual disturbances Schizotypal Personality Few friends Odd attire, behavior, and beliefs Social anxiety Treatment: psychotherapy, antipsychotics

Which of the following tests is initially required weekly to monitor patients who are prescribed clozapine? A. Complete blood count B. Complete metabolic panel C. Hemoglobin A1C D. Lipid panel

A. Complete blood count Antipsychotics are used in the treatment of mental disorders that present with psychosis, which is defined as a loss of contact with reality. Clinically, psychosis presents as delusions, hallucinations, thought disorganization, and agitation or aggression. Antipsychotics work by blocking the postsynaptic dopamine D2 receptors. The medications also affect serotonin, alpha-1, histamine, and muscarinic receptors, accounting for the majority of their side effects. The first-generation antipsychotics, also called typical antipsychotics, are more likely to present with extrapyramidal side effects (akathisia, rigidity, bradykinesia, acute dystonic reactions). This class includes haloperidol, pimozide, and chlorpromazine, among others. Second-generation antipsychotics (atypical antipsychotics) include medications such as risperidone, lurasidone, olanzapine, quetiapine, and clozapine. The side effect profile of this newer medication class includes sedation, weight gain with related metabolic effects (e.g., insulin resistance), hypotension, anticholinergic symptoms, and sexual dysfunction. Rare but life-threatening side effects that can occur with either generation of antipsychotics are tardive dyskinesia, neuroleptic malignant syndrome, seizure, and agranulocytosis. Clozapine is a second-generation antipsychotic particularly notorious for causing agranulocytosis, which is the severe depletion or complete absence of granulocytes (neutrophils). Agranulocytosis can be fatal due to the increased risk of infection and decreased immune response. It is most common in the first few months of treatment and nearly 1% of all patients will develop agranulocytosis in the course of their treatment with clozapine. It is used only in the management of treatment-resistant schizophrenia, and prescribers must be approved through a Risk Evaluation Mitigation Strategy called the Clozapine REMS Program. Severe neutropenia is defined as an absolute neutrophil count less than 500/mm3. A baseline complete blood count must be completed prior to initiating the drug and must demonstrate an absolute neutrophil count of 1,500/mm3 or more. With a normal baseline level, patients are monitored weekly from drug initiation to six months, biweekly from six months to one year, and monthly thereafter. Treatment may continue until absolute neutrophil count falls below 1,000/mm3, at which time the drug should be discontinued and hematology consultation should be sought. Clozapine Agranulocytosis Seizures Myocarditis Increased mortality in older patients with dementia-related psychosis

A 28-year-old woman presents to the clinic with persistent hemiparesis that has caused her to be unable to work. Extensive medical workup has not identified an etiology of her symptoms. The patient is not intentionally producing her symptoms. What is the most likely diagnosis? A. Conversion disorder B. Factitious disorder C. Illness anxiety disorder D. Malingering

A. Conversion disorder Conversion Disorder, Functional Neurologic Symptom Disorder DSM-5 criteria ≥ 1 symptom(s) of altered sensory function or altered voluntary motor function Not consistent with recognized neurological or medical conditions Patient's symptoms are not better explained by another medical condition Symptoms cause Significant distress Impairment in functioning OR Need for medical evaluation Treatment includes patient education and developing a therapeutic alliance (first-line), physical therapy for motor Sxs, cognitive behavioral therapy (CBT) for other somatic symptoms Conversion disorder (functional neurologic symptom disorder) is characterized by neurologic symptoms that are inconsistent with known neurologic disease yet cause significant distress and impair social, occupational, or other important functions. The name conversion disorder comes from the idea that individuals with this diagnosis convert psychological concerns into neurologic symptoms. Functional neurologic symptom disorder is common and has a poor prognosis. Symptoms and signs of conversion disorder may include a variety of neurologic complaints, including nonepileptic seizures, weakness or paralysis, speech disturbances, abnormal movement, globus sensation, sensory symptoms, visual symptoms, or cognitive symptoms. The symptoms may present as an acute or chronic complaint and may be intermittent or constant. Patients with functional neurologic symptom disorder often have comorbidities marked by unexplained symptoms, such as irritable bowel syndrome or chronic fatigue syndrome.

A 25-year-old woman presents to the clinic with excessive worrying that she states has been occurring for years. She states that her excessive worrying is often about minor problems and causes insomnia, daytime fatigue, and difficulty concentrating. Her thyroid-stimulating hormone level is 1.0 mIU/L. She denies taking any medications or using any illicit substances. Which of the following is the most likely diagnosis? A. Generalized anxiety disorder B. Hyperthyroidism C. Panic disorder D. Phobia

A. Generalized anxiety disorder Generalized Anxiety Disorder (GAD) Excessive worry Persistent symptoms Difficult to control feelings Symptoms last over 6 months Treat with CBT, SSRI

An 8-year-old boy presents to the clinic with his mother after his teacher was concerned about disruptive behavior in class. The teacher sent home a note saying that the patient is easily distracted, has difficulty maintaining focus, becomes bored easily, is impatient, talks nonstop, interrupts conversations, and blurts out inappropriate comments. These behaviors are affecting his ability to learn and are disrupting the rest of the class. The teacher is concerned about a potential attention-deficit disorder. Which part of his history would be consistent with attention-deficit/hyperactivity disorder? A. He displays these behaviors at home B. He has no respect for other people's property C. He has trouble communicating with others D. He is vindictive

A. He displays these behaviors at home Attention-deficit/hyperactivity disorder (ADHD) is a triad of impulsivity, inattention, and hyperactivity. Per the Diagnostic and Statistical Manual of Mental Disorders, volume 5 (DSM-5), there are three subtypes of ADHD: hyperactive-impulsive, inattentive, and combined. There are 18 recognized hyperactive and inattentive symptoms, and the child must present with at least six symptoms in one category for diagnosis. Additionally, the symptoms must be present before the age of 12, must be present in at least two different settings (home and school, typically), and must be present for more than six months. Treatment for ADHD includes behavior modification and pharmacologic therapy. There are stimulant and nonstimulant medications, both are appropriate therapies for children. Stimulant medications include methylphenidate and amphetamine-dextroamphetamine. Side effects of stimulants include anxiety, hypertension, tachycardia, growth delays, and loss of appetite. Nonstimulants, such as atomoxetine, have similar efficacy and side effects to stimulant medications, but the side effects occur less often.

A 42-year-old man presents to his primary care provider for a routine physical exam. He has a history of gout and would like refills for indomethacin. He reports his gout has mostly been under control due to changes in diet, but he has occasional flare-ups. His physical exam is unremarkable. As per routine, he is screened for "risky alcohol use." Which of the following responses would indicate this patient meets criteria for risky alcohol use? A. More than 14 standard drinks per week on average B. More than 2 drinks on any given day C. More than 3 drinks on any given day D. More than 7 standard drinks per week on average

A. More than 14 standard drinks per week on average Risky alcohol use is defined as alcohol consumption that puts individuals at risk for health consequences. Medical consequences of risky alcohol use include bone marrow suppression, cardiac symptoms, central or peripheral neurologic symptoms, electrolyte disturbances, gastrointestinal symptoms, hypertension, and elevated liver enzymes. Relevant behavioral, psychiatric, and social consequences are frequent comorbidities and include anxiety, depression, trauma, injury, substance use disorders, suicidality, and social and legal problems. Based on the available epidemiological evidence, men under age 65 are at increased health risk if they consume more than 14 standard drinks per week (where a standard drink is equivalent to 5 oz of wine, 12 oz of beer, or 1.5 oz of 80 proof spirits) on average, or more than 4 drinks on any day. Women and adults 65 years and older are at increased health risk if they consume more than 7 standard drinks per week on average or more than 3 drinks on any day. Certain population groups, for example pregnant women, individuals with alcohol-associated injuries, or individuals with sexually transmitted diseases, may be at increased risk even when consuming smaller, regular amounts of alcohol.

A 22-year-old woman has had uncontrolled episodes of binge eating and self-induced vomiting at least twice weekly. For how long does this behavior need to occur to be diagnosed as bulimia nervosa? A. Three months B. Three weeks C. Two months D. Two weeks

A. Three months Bulimia Nervosa Patient with a history of binge eating followed by purging via forced vomiting, purging via laxative misuse, driven exercise, or fasting PE will show dental erosions and callused knuckles Diagnosis requires at least one episode of binge eating with inappropriate compensatory behavior per week for a minimum of 3 months in a patient whose self-evaluation is unduly influenced by body shape or weight Treatment is cognitive behavioral therapy, SSRIs, or both

Which of the following is a risk factor for the development of antisocial personality disorder? A. Conduct disorder B. Female sex C. Incarceration D. Mood disorder in first-degree relative

A. conduct disorder Antisocial Personality Disorder Adults over 18 years old but symptoms occurring since age 15 Evidence of conduct disorder before the age of 15 May have a history of child abuse Disregard for others Violate laws and social norms Treat with psychotherapy Clinical manifestations of antisocial personality disorder include manipulation, exploitation, or deception of others to get what they want, impulsivity, reckless behavior, social and financial irresponsibility, physical aggression, arrogance, and lack of empathy and remorse. Differential diagnoses include substance use disorder, post-traumatic stress disorder, depression, anxiety disorder, borderline personality disorder, narcissistic personality disorder, intermittent explosive disorder, and medical disorders affecting the brain, such as traumatic brain injury, brain tumor, temporal lobe epilepsy, and stroke. The foundation of treatment for all personality disorders is psychotherapy. Medication is not curative for personality disorders and should be used as an adjunct to psychotherapy or when there is a co-occurring mood disorder.

Which of the following findings is most supportive of a current diagnosis of antisocial personality disorder? A. A child setting the school on fire B. A man raping multiple women without remorse C. Using physical appearance to draw attention to oneself D. Viewing others as either all good or all bad

B. A man raping multiple women without remorse Antisocial Personality Disorder Adults over 18 years old but symptoms occurring since age 15 Evidence of conduct disorder before the age of 15 May have a history of child abuse Disregard for others Violate laws and social norms Treat with psychotherapy

A 24-year-old man is diagnosed with schizophrenia and is admitted to an inpatient psychiatric facility. He is treated with intramuscular haloperidol. While he is being treated, he describes a sensation of inner restlessness and an inability to sit still. Which common adverse effect of first-generation antipsychotics is he experiencing? A. Acute dystonic reaction B. Akathisia C. Orthostatic hypotension D. Tardive dyskinesia

B. Akathisia First-generation (typical) antipsychotics are used in the treatment of acute psychosis and chronic psychiatric conditions. This class of medication works primarily as central dopamine receptor antagonists. However, slight interactions with histamine, acetylcholine (muscarinic), alpha-1-adrenergic, and serotonin receptors may occur and are responsible for some of the adverse effects. First-generation antipsychotics include fluphenazine, haloperidol, loxapine, chlorpromazine, and thioridazine. Haloperidol is the most widely studied and frequently used first-generation antipsychotic. Common conditions treated with antipsychotics include bipolar disorder, schizophrenia, delusional disorder, unipolar depression with psychosis, and acute illicit drug-induced psychosis. Adverse effects of first-generation antipsychotics include extrapyramidal side effects, tardive dyskinesia, findings similar to metabolic syndrome, hyperprolactinemia, orthostatic hypotension, anticholinergic side effects, sexual dysfunction, QT prolongation, and neuroleptic malignant syndrome. The presence of extrapyramidal side effects and tardive dyskinesia distinguish the side effect profile of first-generation from second-generation antipsychotics. Extrapyramidal adverse effects include akathisia (inner feeling of restless but inability to sit still), acute dystonic reactions, and Parkinsonian symptoms. These effects occur due to interference with dopamine transmission in the nigrostriatal pathway, which produces symptoms similar to those seen in Parkinson disease. Akathisia is the most common form of extrapyramidal symptoms. Patients often describe it as restlessness with a compelling urge to move and inability to sit still. Parkinsonian syndrome includes mask-like facies, resting tremor, cogwheel rigidity, shuffling gait, and psychomotor retardation. Until the medication dose has been stable for at least two weeks, patients on first-generation antipsychotics should be assessed weekly to check for extrapyramidal adverse effects or tardive dyskinesia. Patients on stable medication dosages should be assessed every three to six months. The management of extrapyramidal adverse effects and tardive dyskinesia often begins with considering dose reduction or switching to a different antipsychotic (often second-generation antipsychotics). However, many patients cannot discontinue antipsychotics and some patients may not be able to switch to a second-generation antipsychotic. Furthermore, dose reductions may not be possible due to exacerbation of the underlying psychiatric condition. Therefore, pharmacologic therapy is often required to treat the adverse effects. Patients with mild tardive dyskinesia associated with anxiety can be treated with low doses of a benzodiazepine, such as clonazepam. Akathisia may be treated with benztropine, beta-blockers (propranolol), or benzodiazepines. The management of metabolic syndrome adverse effects also involves attempting to switch antipsychotics or lower the dose. Additional interventions include lifestyle modifications and pharmacologic treatment of the underlying conditions, such as dyslipidemia and diabetes mellitus. Tobacco cessation should also be emphasized because it can help reduce the risk of vascular disease. Hyperprolactinemia can manifest as elevated plasma prolactin, galactorrhea or menstrual abnormalities in women, gynecomastia in men, and sexual dysfunction in women and men. Patients with hyperprolactinemia secondary to an antipsychotic are usually managed by switching to a different antipsychotic. However, metformin may be able to help reduce antipsychotic induced hyperprolactinemia. Neuroleptic malignant syndrome presents with fever, muscle rigidity, mental status changes, autonomic instability, and is generally accompanied by rhabdomyolysis and an increase in serum creatine kinase. Neuroleptic malignant syndrome is a rare potentially fatal condition, but there are case reports of it occurring with each first-generation antipsychotic.

After a thorough evaluation, your new patient is diagnosed with specific phobia related to flying. She has agreed to start cognitive-behavioral therapy, but because flying is required as part of her work, you also decide to prescribe a short-term rescue medication. Which of the following medications would you prescribe as a first-line treatment? A. Escitalopram B. Lorazepam C. Modafinil D. Quetiapine

B. Lorazepam Specific phobia is an anxiety disorder characterized by intense fear of an object or situation and is associated with avoidance behavior. It can cause significant embarrassment, distress, functional impairment, and disability, yet only a small percentage of patients seek treatment. The treatment of specific phobia centers around cognitive-behavioral therapy with exposure, but in certain instances, pharmacologic therapy is also indicated.

A 22-year-old woman presents to your clinic with concerns about her mental health. She says that last month she felt very depressed, was sleeping 12 hours per night, and missed several days of work. Last week, she suddenly felt significantly more energetic, was sleeping three to four hours per night, and was gambling excessively. During your interview, she talks very quickly and is easily distracted. She says that her family has expressed concern because she told them last night that she was having thoughts of suicide. Which of the following is the most likely diagnosis? A. Attention deficit hyperactivity disorder B. Bipolar disorder C. Borderline personality disorder D. Schizoaffective disorder

B. Bipolar disorder The two types of bipolar disorder are differentiated by the severity of mania. Both types have episodes of severe depression. The mania of bipolar I is severe and can include psychosis and significant impairment in functioning. The mania of bipolar II disorder is a less severe form called hypomania, which does not include psychotic symptoms or a significant impact on daily functioning. There is a genetic component to bipolar disorder with individuals having a first-degree relative with the disorder being seven times more likely to develop it than the general population. Bipolar disorder affects men and women equally, although certain aspects of the disorder, such as rapid cycling and bipolar II, are seen more commonly in women. There is a wide age range for the development of bipolar disorder, from childhood to age 50 years, although most cases develop in the late teens or early 20s. Symptoms of bipolar disorder can present similarly to other mental health disorders and diagnosis can sometimes take years to clarify. Treatment includes determining the need for inpatient care and using mood-stabilizing medications and psychotherapy, which help support the individual with bipolar disorder but do not cure the disease.

A 24-year-old woman presents to her counselor due to relationship difficulties. She frequently engages in self-mutilating behavior and reports a fear of abandonment. The counselor talks with her friends who state that one day she says she loves them and the next day she says she hates them. Which personality disorder does this patient most likely have? A. Antisocial B. Borderline C. Histrionic D. Paranoid

B. Borderline Borderline Personality Disorder (BPD) Cluster B personality disorder F > M Splitting Unstable mood and relationships Self-mutilation, increased suicide risk Psychotherapy The recommended treatment of all patients with borderline personality disorder is evidence-based psychotherapy. Pharmacologic therapy may be used to treat specific symptoms. Patients with cognitive-perceptual symptoms may benefit from second-generation antipsychotics, such as quetiapine. Cognitive-perceptual symptoms include disturbed identity, paranoia, and hallucinations. Patients with borderline personality disorder with severe impulsivity (e.g., interpersonal hostility and aggression, self-injury, or recklessness) should be treated with a mood stabilizer (e.g., lithium, valproate, carbamazepine) rather than an antipsychotic. Patients with borderline personality disorder and severe affective dysregulation (e.g., depressed mood, anxiety) should also be treated with mood stabilizers. The treatment of patients with borderline personality disorder and comorbid unipolar depression or anxiety should focus primarily on treating borderline personality disorder, as this approach is more likely to lead to improvement of both disorders. However, in patients with a co-occurring substance use disorder, treatment of the substance use disorder should take precedence.

A 25-year-old woman presents to the emergency department after a suicide attempt. After the patient is stabilized and admitted to the psychiatric floor, a psychiatric provider evaluates her. She discloses to the provider that she has been struggling with depression and chronic alcohol use and complains that all of her friends are "terrible for not coming to visit her in the hospital." She states they constantly abandon her. Based on the patient's presentation, which of the following is the most likely diagnosis? A. Bipolar disorder B. Borderline personality disorder C. Histrionic personality disorder D. Major depressive disorder

B. Borderline personality disorder Borderline personality disorder is characterized by a pattern of mood instability that is associated with intense, but unstable, interpersonal relationships. The patient is commonly impulsive, prone to anger, and frequently threatens suicide or self-harm. Also, patients with borderline personality disorder are sensitive to critique, rely on validation from others, and are afraid of abandonment. Comorbid conditions often include alcohol use, substance use, and binge eating. There is no test to diagnose borderline personality disorder other than meeting the necessary personality traits described in the Diagnostic and Statistical Manual of Mental Disorders, volume 5 (DSM-5). Treatment for borderline personality disorder can be challenging , and first-line therapy is psychotherapy, dialectical behavioral therapy, or cognitive behavioral therapy. Mood stabilizers, specifically lithium and valproic acid, are helpful in controlling anger outbursts and impulsivity.

A 30-year-old woman with a known history of anxiety presents to her provider to have her anxiety medications refilled. She has been using more of her medications recently because she has had some new symptoms. In particular, she is experiencing intrusive thoughts and feels compelled to do things to quiet the thoughts, which sometimes drives her to take her anxiety medicine. These unwanted thoughts are starting to interfere with her ability to get to work on time and focus on work while she is there. Which of the following would be most consistent with the suspected diagnosis? A. Avoidance of public transportation or crowds due to fear of an inability to escape B. Checking to make sure all appliances and lights are off before leaving a room due to concern for fire C. Checking weight repeatedly and adjusting diet due to fear of gaining weight D. Impulse to pick skin and repeated attempts to stop the habit

B. Checking to make sure all appliances and lights are off before leaving a room due to concern for fire Effective treatment options for OCD include cognitive behavioral therapy and serotonergic antidepressants. Obsessive-Compulsive Disorder Recurrent thoughts (obsessions), behaviors (compulsions) Difficult to control Disruption of daily living Need to perform rituals Yale-Brown Obsessive-Compulsive Scale Treat with medications and therapy

Which of the following characteristics would be most definitive in a diagnosis of anorexia nervosa? A. A disturbance of body image B. Loss of weight to more than 15% below the expected body weight C. Repeated purging by vomiting after eating D. The absence of two consecutive menstrual cycles in a female

B. Loss of weight to more than 15% below the expected body weight Anorexia Nervosa Three core features -Restriction of energy intake relative to requirements leads to significantly low body weight -Intense fear of gaining weight or persistent behavior interfering with weight gain -Disturbance in body image Two types -Restricting -Binge-eating and purging type Recommended interventions -Determination of setting -Medical stabilization -Nutritional rehabilitation -Psychotherapy -For youth, best evidence is for family-based therapy, including parental management of ongoing weight restoration

Which of the following is indicated for treatment of obsessive-compulsive disorder? A. Amitriptyline B. Clomipramine C. Doxepin D. Imipramine

B. Clomipramine Obsessive-compulsive disorder is a mental health condition with a lifetime prevalence of 2-3% of all individuals. The prevalence of obsessive-compulsive disorder does not vary significantly between race or gender. However, men are more likely to have a comorbid tic disorder. Childhood streptococcal infection followed by Pediatric Autoimmune Neuropsychiatric Disorders Associated with Streptococcal Infections (PANDAS) is a risk factor for development of obsessive-compulsive disorder. According to the Diagnostic and Statistical Manual of Mental Disorders, 5th Edition, obsessive-compulsive disorder occurs when an individual suffers functional impairment due to time-consuming (>1 hour) intrusive thoughts ("obsessions") or time-consuming repetitive behaviors that the patient feels compelled to perform ("compulsions"). Clomipramine is a tricyclic antidepressant that is indicated for use in obsessive-compulsive disorder. Like all tricyclic antidepressants, clomipramine inhibits the reuptake of norepinephrine while blocking N-methyl-D-aspartate and sodium channel receptors. Clomipramine is also a strong serotonin reuptake inhibitor and treats obsessive-compulsive disorder by increasing the availability of serotonin (a neurotransmitter) in the brain. Clomipramine is often used to augment selective serotonin reuptake inhibitor therapy, which is the first-line treatment for obsessive-compulsive disorder. Combining use of clomipramine with a selective serotonin reuptake inhibitor provides enough serotonergic activity to treat obsessive-compulsive disorder and avoids the gastrointestinal distress seen with a high dose selective serotonin reuptake inhibitor monotherapy, which is often intolerable for patients. Clomipramine (like other tricyclic antidepressants) has strong anticholinergic properties and should be avoided in patients with urinary or gastrointestinal retention or glaucoma. Additionally, tricyclic antidepressants prolong QT intervals and may produce sinus tachycardia or fatal ventricular arrhythmias. Unlike selective serotonin reuptake inhibitors, clomipramine overdose can be used as a method to complete suicide and should not be given to patients displaying suicidal ideation or behavior.

A 35-year-old man presents complaining of lacking self-confidence. At work, he finds it very difficult to initiate any projects. He would volunteer to work extra on the weekends without pay. He always avoids confrontation for fear of losing approval. He broke up with "the love of my life" two days ago and has now met "the new love of my life." He has difficulty deciding what to eat for lunch without asking for an excessive amount of advice. Which of the following personality disorders best fits with his behavior? A. Avoidant personality disorder B. Dependent personality disorder C. Histrionic personality disorder D. Obsessive-compulsive personality disorder

B. Dependent personality disorder Individuals with dependent personality disorder have a pervasive and excessive need to be taken care of leading to submissive behaviors, clinging behaviors, and fears of separation. The Diagnostic and Statistical Manual of Mental Disorders, 5th edition criteria for diagnosing dependent personality disorder include five or more of the following: difficulty making everyday decisions, needing others to assume responsibility for major areas of their lives, lack of self-confidence, immediately seeking a new relationship when a close relationship ends, difficulty disagreeing with others, difficulty initiating projects, helpless when alone, and preoccupation with fear of being left to take care of himself. Dependent personality disorder is classified as cluster C, a group of disorders that make individuals appear anxious or fearful and commonly begins by early adulthood. Early referral to a mental health professional is very important in managing dependent personality disorder. The first-line treatment of dependent personality disorder is psychotherapy with or without antidepressants or mood stabilizers.

A 47-year-old man with a psychiatric history of bipolar type 2 disorder, remote substance use disorder, and three recent psychiatric hospitalizations presents to the emergency department within 48 hours of his most recent discharge, complains "I want to kill myself...your dumb social worker sent me to an awful sober living facility!" He becomes belligerent when asked to elaborate on his plan for suicide. Then, on examination, the patient quickly shifts from anger to crying, stating, "Finally! A doctor that I can trust!" When asked to estimate a duration for his current suicidal ideation, the patient states "I am always a wreck. That never changes." On mental status exam, the patient displays labile affect, but his thought process is linear and without pressured speech or flight of ideas. Which of the following is the treatment of choice for this patient? A. Aripiprazole B. Dialectic behavioral therapy C. Sober living home D. Valproate

B. Dialectic behavioral therapy Dialectic behavioral therapy is the treatment of choice for borderline personality disorder. Dialectic behavioral therapy promotes distress tolerance, interpersonal effectiveness, and long-term emotional stability. Patients with borderline personality disorder suffer from continual emotional dysregulation, which can manifest as a constellation of five or more of the following symptoms: pervasive fear of abandonment, unstable relationships, unstable self-image, impulsivity, recurrent suicidal behavior, lability, feelings of emptiness, inappropriate anger, stress-related paranoia or dissociative symptoms. Borderline personality disorder may occur comorbidly with bipolar disorder, however, it is often misdiagnosed. Patients with borderline personality disorder are not candidates for mood stabilizers or antipsychotics. There is no acute treatment for borderline personality, and hospitalization (although often sought by these patients) is of little benefit. Borderline personality disorder should be referred to outpatient services for long-term dialectic behavioral therapy.

A 27-year-old woman recently suffered the trauma of the birth of a stillborn baby. When questioned about the birth she has no recollection of the events. Which of the following is the most likely diagnosis? A. Depersonalization/derealization disorder B. Dissociative amnesia C. Dissociative fugue D. Dissociative identity disorder

B. Dissociative amnesia Dissociative amnesia is an inability to recall important personal information, which is inconsistent with normal forgetfulness. In most cases, the amnesia develops after an especially stressful or traumatic incident or experience. Dissociative amnesia occurs equally in both men and women. Sexual abuse, witnessed suicides or homicides, or violent trauma has all been linked with occurrences of dissociative amnesia.

Which of the following neurotransmitters is affected most by first-generation antipsychotic medication used to treat patients with schizophrenia? A. Acetylcholine B. Dopamine C. Glutamate D. Norepinephrine

B. Dopamine Schizophrenia Two or more symptoms Delusions, hallucinations, disorganized speech, grossly disorganized or catatonic behavior, negative symptoms Brief psychotic disorder: < 1 month Schizophrenia: > 6 months Schizophreniform disorder: 1-6 months Schizoaffective disorder: psychosis + mania or depression Antipsychotic medications are the treatment of choice. First-generation, or typical, antipsychotics typically have a worse side effect profile than newer second-generation, or atypical, antipsychotics.

Which of the following is first-line treatment for anorexia nervosa? A. Fluoxetine B. Nutritional rehabilitation C. Olanzapine D. Orlistat

B. Nutritional rehabilitation Psychopathology regarding body image and food generally does not respond to pharmacologic therapy. However, acutely ill patients who are not gaining weight despite standard treatment (psychotherapy and nutritional rehabilitation) are candidates for adjunctive pharmacotherapy. The recommended pharmacologic agent for these patients is the second-generation antipsychotic olanzapine. Patients who have recovered from an episode of anorexia nervosa should be treated with maintenance psychotherapy without pharmacologic therapy. Howev

A 24-year-old man presents to the psychiatry clinic with auditory hallucinations and delusions for the past seven months. On exam, the psychiatrist notes disorganized speech and alogia. He is diagnosed with schizophrenia and started on a first-generation antipsychotic. What is the mechanism of action of this medication? A. Dopamine agonist B. Dopamine antagonist C. Serotonin and dopamine antagonist D. Serotonin reuptake inhibitor

B. Dopamine antagonist The mechanism of action of first-generation antipsychotics (also known as neuroleptics or typical antipsychotics) is postsynaptic blockade of dopamine D2 receptors. They are central dopamine antagonists. Furthermore, each individual first-generation antipsychotic has distinct effects on some of the following receptors: serotonin, alpha-1-adrenergic receptors, histamine receptors, and muscarinic receptors. The additional receptors that each antipsychotic interacts with causes some of the adverse effects. First-generation antipsychotics include fluphenazine, haloperidol, loxapine, chlorpromazine, and thioridazine. Haloperidol is the most widely studied and frequently used first-generation antipsychotic. Most of these medications are dosed orally, but intramuscular injections are available for acute indications, such as an irritable patient not responding to verbal instructions. Important adverse effects of first-generation antipsychotics include extrapyramidal side effects, tardive dyskinesia, hyperprolactinemia, QT prolongation, sudden death, neuroleptic malignant syndrome, increased mortality in older patients with dementia, anticholinergic adverse effects, sedation, sexual dysfunction, and orthostatic hypotension. The association of first-generation antipsychotics with movement disorders is the primary difference between first- and second-generation antipsychotics. Extrapyramidal adverse effects include akathisia (inner feeling of restless but inability to sit still), rigidity, bradykinesia, tremor, and acute dystonic reactions.

A man presents to various healthcare settings falsely stating he has abdominal pain. He induces vomiting and uses diuretics to attempt to falsify his electrolyte concentrations. There is not an obvious source of external reward for his behaviors. What is the most likely diagnosis? A. Conversion disorder B. Factitious disorder C. Malingering D. Somatic symptom disorder

B. Factitious disorder The diagnosis of factitious disorder imposed on self should be discussed with patients in an accurate yet compassionate way. It is important to emphasize that the patient needs help and to avoid expressing anger, acting judgmental, or taking retaliatory action. Embarrassment can be minimized by discussing the entire differential diagnosis and not just factitious disorder. However, most patients diagnosed with factitious disorder deny simulating or inducing their illness. Psychotherapy is first-line treatment rather than pharmacotherapy or no treatment. Supportive psychotherapy or cognitive-behavioral therapy is typically the recommended modality of psychotherapy. Patients with factitious disorder and other comorbid psychiatric conditions should be treated with standard first-line treatments of those conditions. The prognosis for factitious disorder imposed on self is poor, and recovery appears to be infrequent.

A 20-year-old woman presents with an excessive need to wash her hands for fear that she may have been in contact with contaminated objects. She spends more than one hour per day washing her hands, to the extent that this behavior is causing some social impairment. Which of the following therapies is the most appropriate for this patient? A. Clomipramine B. Fluoxetine C. Lithium D. Venlafaxine

B. Fluoxetine The diagnosis of OCD is clinical and based on the presence of obsession, compulsions, or both that are time-consuming (greater than one hour per day) or cause severe impairment or distress. Exposure and ritual prevention therapy and pharmacotherapy are the mainstay treatments for OCD. Exposure and ritual prevention therapy involves gradually exposing patients to situations that trigger anxiety-provoking obsessions and rituals while asking them not to perform their rituals. By doing so, anxiety associated with the obsessions diminishes through habituation. First-line pharmacotherapy for treating OCD is selective serotonin reuptake inhibitors (SSRIs, including sertraline, paroxetine, fluoxetine, citalopram). Patients often require higher doses than are typically needed for anxiety and depressive disorders. For severe cases of OCD, exposure and ritual prevention therapy in addition to pharmacotherapy is the best treatment option

A 30-year-old woman presents to the clinic for evaluation for breast cancer. She performs three breast self-examinations per day but has never had any symptoms or signs of breast cancer. She says that her grandmother was diagnosed with breast cancer at 80 years of age, and so she knows she will get breast cancer one day. Which of the following is the most likely diagnosis? A. Generalized anxiety disorder B. Illness anxiety disorder C. Panic disorder D. Somatic symptom disorder

B. Illness anxiety disorder Illness Anxiety Disorder Hypochondriasis Preoccupied with serious illness, despite negative exam or testing Evaluate for other medical diagnosis Management is CBT, psychotherapy, antidepressants

A 27-year-old man presents to the clinic with depressed mood associated with fatigue, decreased concentration, hypersomnia, increased appetite, and feelings of guilt for the past three weeks. He is diagnosed with unipolar depression, and the clinician starts him on bupropion. Which of the following is an adverse effect of bupropion? A. Increased risk of eating disorders B. Increased risk of seizure C. Sexual dysfunction D. Weight gain

B. Increased risk of seizure Bupropion MOA: dual norepinephrine and dopamine reuptake inhibitor FDA-approved indications -Major depressive disorder -Seasonal affective disorder -Nicotine dependence Lower risk -Weight gain -Sexual side effects -Induction of hypomania or mania Higher risk -Seizures, particularly if other risk factors or exceeding dosing guidelines -Allergic reactions: anaphylactic, Stevens-Johnson syndrome

A 24-year-old man has been diagnosed with bipolar disorder type I and is being started on a new medication that is known to cause tremors, hypothyroidism, weight gain, and nephrogenic diabetes insipidus. Which of the following medications is being started? A. Carbamazepine B. Lithium C. Olanzapine D. Valproic acid

B. Lithium The mainstay of treatment of bipolar disorder type I is mood stabilizers such as lithium, valproic acid, olanzapine, and carbamazepine. Lithium has a narrow therapeutic window, and plasma levels must be monitored every four to eight weeks. Prior to initiating therapy with lithium, renal function should be assessed, as lithium is cleared by the kidneys. Thiazide diuretics reduce the renal clearance of lithium by 25%. Hence, dosage adjustment is warranted. Adverse effects of lithium include neurological symptoms (tremor, ataxia, hyperactivity, aphasia, and choreoathetosis), hypothyroidism, nephrotic diabetes insipidus, edema (weight gain), and bradycardia-tachycardia syndrome. Overdose with lithium leads to lithium toxicity, which is characterized clinically by diarrhea, vomiting, tremor, muscle weakness, slurred speech, aphasia, leukocytosis, and prolonged QT interval. For acute mania, second-generation antipsychotics (risperidone, aripiprazole, quetiapine, ziprasidone) or benzodiazepines are effective. Haloperidol may be added if psychotic features are present. Acute depressive episodes can be treated with selective serotonin receptor inhibitors or quetiapine. Antidepressants may precipitate mania. Refractory cases should be treated using secondary therapies, including electroconvulsive therapy, monoamine oxidase inhibitors (MAOIs), and tricyclic antidepressants (TCAs). MAOIs and TCAs can cause rapid cycling between mood states and should be used with caution.

A 32-year-old man presents to the clinic accompanied by his girlfriend. She describes episodes wherein the patient checks every door in their house exactly three times before they leave to ensure they are all closed. She states this happens every time they leave their house. When questioned about this behavior, the patient replies he "feels a need" to check every door and cannot leave the house without fulfilling the need. Which of the following disorders is the most likely diagnosis? A. Generalized anxiety disorder B. Obsessive-compulsive disorder C. Obsessive-compulsive personality disorder D. Tic disorder

B. Obsessive-compulsive disorder Treatment should involve cognitive-behavioral therapy with exposure and response prevention. A selective serotonin reuptake inhibitor (e.g., fluoxetine, paroxetine, citalopram) may be initiated if therapy is unavailable, ineffective, or unwanted. Successful treatment of obsessive-compulsive disorder usually requires high doses of this class of medications, with continuation at therapeutic range for a minimum of six weeks before determining efficacy. Failure of one medication in this class does not preclude treatment with another, and a different selective serotonin reuptake inhibitor may be started. Venlafaxine, a selective norepinephrine reuptake inhibitor, and clomipramine, a tricyclic antidepressant, have also shown promise in treating this disorder and may be initiated as second-line agents. Families may learn to accommodate compulsions, which leads to an overall poor treatment response and also increases the family burden and reduces the quality of life among family members.

Which of the following is the strongest risk factor for suicide? A. Age group 30 - 50 years B. Previous suicide attempt C. Psychiatric disorders D. Sexual minority

B. Previous suicide attempt Suicide Protective factors: marriage, pregnancy RFs (SAD PERSONS): Sex (male), Age (teenager or elderly), Depression, Previous attempt, Ethanol/drug use, Rational thinking loss, Separated, divorced, or widowed, Organized plan, No Social support, Stated future attempt Most suicides involve firearms Most attempted suicides involve ingestions (MC: antidepressants) All overdose patients: obtain acetaminophen level

A 26-year-old man presents with major depressive episodes, delusions, and hallucinations. He started hearing voices six months ago, and his depressed mood has been present for the past three weeks. Physical exam reveals no abnormality. Laboratory studies and urine drug screens demonstrate no abnormalities. Which of the following is the most likely diagnosis? A. Delusional disorder B. Schizoaffective disorder C. Schizophrenia D. Schizophreniform disorder

B. Schizoaffective disorder Schizoaffective disorder is characterized by psychosis and marked mood symptoms. It meets the criteria for a mood disorder (major depressive episode, manic episode, or mixed episode) and schizophrenia (delusions, hallucinations, disorganized thinking or behavior, and catatonia). Schizoaffective disorder carries a better prognosis than schizophrenia but a worse prognosis than a mood disorder. Men and women are equally affected, although men have an earlier age of onset. Patients are often misdiagnosed with schizophrenia, depression, bipolar disorder, or a mood disorder with psychotic features. The presence of delusions or hallucinations lasting more than two weeks distinguishes schizoaffective disorder from a mood disorder with psychotic features. DSM-5 diagnostic criteria for schizoaffective disorder require the presence of a major mood disorder (either depression or mania), psychosis for two or more weeks, the coexistence of a mood disorder and schizophrenia, and the absence of drug-induced symptoms. The treatment of schizoaffective disorder requires a comprehensive regimen, including pharmacotherapy, psychotherapy, and community support. The first-line pharmacotherapy for schizoaffective disorder is second-generation antipsychotics (risperidone, olanzapine, quetiapine, ziprasidone, aripiprazole, and asenapine). Once positive symptoms are adequately treated, antidepressants may be given if depression is present. Mood stabilizers, such as lithium and valproic acid, can treat mood symptoms. Psychotherapy helps patients to understand and manage their illness.

What medications can be used to treat acute akathisia?

Benztropine, benzodiazepines, or beta-blockers.

A 35 y/o male is brought in by his wife because he has been taking out various loans to start a few small businesses. Over the past 2 weeks, he comes home at 3am from work and leaves at 6am and often compares his business ventures to those of Bill Gates. On exam, he is easily distracted, speaks quickly and says he has to get home as he is close to discovering "the most important invention of the 21st century". In the past he has a few episodes in which he felt hopeless and tried to commit suicide.

Bipolar I In this case, he has delusions of grandiosity making it a full mania

A 34-year-old woman presents to the clinic for evaluation after her family has raised concern over her behavior for the past three months. She states that she discovered the cure for cancer and will soon be famous. Her family reports that she has never had a scientific interest before and she now spends her time calling reporters instead of attending her job. She denies changes in mood or visual or auditory hallucinations. Which of the following is the most likely diagnosis? A. Bipolar disorder B. Brief psychotic disorder C. Delusional disorder D. Schizophrenia

C. Delusional disorder Delusions Fixed, false belief Persecutory most common Schizophrenia spectrum disorders, mood and other disorders with psychotic features

A 24-year-old woman presents to the clinic with excessive and persistent worry most days for the last six months. She states that the worrying disrupts her sleep, causes irritability, and makes her feel fatigued. Which of the following is the recommended first-line pharmacologic treatment? A. Buspirone B. Clonazepam C. Duloxetine D. Mirtazapine

C. Duloxetine Generalized Anxiety Disorder (GAD) Excessive worry Persistent symptoms Difficult to control feelings Symptoms last over 6 months Treat with CBT, SSRI

A 17-year-old girl who is overweight, presents to the clinic with her mother. Her mother is concerned that she has caught her daughter eating large amounts of food in the middle of the night, and she has found receipts for over-the-counter laxatives in the garbage. What physical exam finding is most likely to be seen with the suspected diagnosis? A. Bradycardia B. Dry skin C. Enamel erosion D. Lanugo hair

C. Enamel erosion Bulimia Nervosa Patient with a history of binge eating followed by purging via forced vomiting, purging via laxative misuse, driven exercise, or fasting PE will show dental erosions and callused knuckles Diagnosis requires at least one episode of binge eating with inappropriate compensatory behavior per week for a minimum of 3 months in a patient whose self-evaluation is unduly influenced by body shape or weight Treatment is cognitive behavioral therapy, SSRIs, or both

A 20-year-old woman presents to the clinic with excessive worrying for the past 7 months. She also reports insomnia, daytime fatigue, feeling on edge, and difficulty concentrating. Basic laboratory testing is performed and normal. Toxicology screen is negative. Which of the following is first-line pharmacologic treatment? A. Alprazolam B. Buspirone C. Fluoxetine D. Hydroxyzine

C. Fluoxetine Cognitive-behavioral therapy is generally provided in 10-15 sessions that each last 60 minutes. Medications with efficacy in the treatment of generalized anxiety disorder include serotonin reuptake inhibitor antidepressants, benzodiazepines, pregabalin, and buspirone. The first-line pharmacologic therapy is serotonin reuptake inhibitors, which can be either a selective serotonin reuptake inhibitor, such as fluoxetine, or a serotonin norepinephrine reuptake inhibitor, such as duloxetine or venlafaxine. Each serotonin reuptake inhibitor has similar efficacy, and it generally takes about four weeks for patients to notice clinical improvement. Buspirone is sometimes used to augment other medications. Benzodiazepines, such as alprazolam, can be used to treat generalized anxiety disorder in patients who have not responded to serotonin reuptake inhibitors and who do not have a history of substance use disorder. Additional medications that are sometimes used but lack proven efficacy include mirtazapine (a sedating antidepressant) and second-generation antipsychotic medications, such as quetiapine. Generalized anxiety disorder is a potentially chronic illness with fluctuating symptom severity over time.

A 60-year-old woman presents complaining of severe abdominal pain. She demands to be seen by a male physician assistant. She is talking loudly on the phone with her friend about a sexual encounter she had the night before. She is provocatively dressed. During the physical examination, she insists that the physician assistant "examine her butt" and constantly tells him how "hot" he is. Which of the following personality disorders describes her behaviors? A. Borderline B. Dependent C. Histrionic D. Narcissistic

C. Histrionic The Diagnostic and Statistical Manual of Mental Disorders, 5th edition criteria for diagnosing histrionic personality disorder include five or more of the following: uncomfortable if not the center of attention, sexually provocative interactions with others, shallow expression of emotions, using physical appearance to draw attention, impressionistic style of speech lacking in details, exaggerated emotionality, suggestibility, and thinking that relationships are more intimate than they are. Histrionic personality disorder is classified as cluster B, a group of disorders that make individuals appear dramatic, emotional, and erratic. Treatment of histrionic personality disorder is with psychodynamic psychotherapy.

A 45-year-old man comes to the office for a check-up. He complains of increasing stool frequency from every other day to every day over the past year. He says that he has colon cancer and would like to be evaluated for treatment. He admits having been worked up for colon cancer by several physicians who could not find the cancer despite him constantly telling them it is there. Family and medical history is noncontributory. Cardiopulmonary examination is normal and abdominal examination shows positive bowel sounds and a soft non-tender non-distended abdomen with no palpable masses. Stool occult blood testing is negative. Which of the following is the most likely diagnosis? A. Conversion disorder B. Factitious disorder C. Illness anxiety disorder D. Malingering E. Pain disorder

C. Illness anxiety disorder Illness anxiety disorder is a DSM-V characterized by misinterpretation of minor bodily symptoms as a serious illness. Patients often self diagnose themselves and do not believe the physician's negative work-up. Preoccupation with fear of death must cause a great deal of distress or interfere with a person's ability to perform important activities, such as work, school activities, or family and social responsibilities. The preoccupation must be present for at least six months. Treatment consists of regular follow-up visits and offering psychotherapy. The patient should not be told that these symptoms are imaginary. Illness anxiety disorder was previously known as hypochondriasis, however this is not a possible diagnosis in the DSM-V. Another similar DSM-V diagnosis to illness anxiety disorder is somatic symptom disorder which is characterized by excessive preoccupation with a chronic symptom.

A 58-year-old homeless man is brought to the emergency department by the police. The police officers mention that they found the patient screaming near a shopping mall. When taken into custody, the officers mention that he was aggressive and kept claiming he was a diplomatic leader from Iceland. When asking the patient about his personal information, he looks apathetic, he has no recollection of who he is, where he is from, or what happened prior to being found by the police. Upon further interrogation, the patient seems confused, disoriented and has to be reminded several times that he is in the hospital. On physical examination, the patient looks malnourished he shows a strong alcoholic odor and facial hyperemia. His temperature is 36.6°C (97.8°F), pulse is 102/min, respirations are 20/min, blood pressure is 130/80 mmHg. The toxicology screen is positive for alcohol. Which of the following is the most likely diagnosis? A. Alcohol use disorder B. Wernicke encephalopathy C. Korsakoff's syndrome D. Hepatic encephalopathy E. Methanol poisoning

C. Korsakoff's syndrome Korsakoff's syndrome is a neurological disorder caused by a lack of thiamine (vitamin B1). This condition is a manifestation of untreated Wernicke encephalopathy. Patients most often present with severe memory loss (anterograde and retrograde amnesia), confabulation (fabricated autobiographical memories), apathy, and lack of insight.

A 25-year-old man presents to the psychiatrist for evaluation. He has a history of a hypomanic episode and several major depressive episodes and is currently experiencing a depressive episode. Which of the following is an appropriate pharmacologic treatment for this individual's depressive episode? A. Duloxetine B. Fluoxetine C. Lamotrigine D. Venlafaxine

C. Lamotrigine The management of bipolar disorder type 2 involves treating hypomanic and depressive episodes and maintenance treatment between episodes. Hypomania is treated with the same medications as mild and moderate mania. The second-generation antipsychotics olanzapine and risperidone are first-line pharmacologic agents for hypomania. However, other reasonable alternatives include lithium or other antipsychotics and anticonvulsants. Lithium or valproate are often combined with an antipsychotic in patients with bipolar disorder type 2 who fail to improve with three to five monotherapy trials. The recommended treatment for acute depressive episodes in patients with bipolar disorder consists of lithium, antipsychotics, or anticonvulsants, such as lamotrigine. Antidepressants, such as fluoxetine, are sometimes used as adjunctive treatment to antipsychotics. However, antidepressants in combination with lithium or valproate have not been proven to be effective. Furthermore, antidepressant monotherapy may precipitate mania. Electroconvulsive therapy is an alternative treatment that can be used in refractory cases and has proven to be effective. The maintenance treatment for bipolar disorder often consists of the medications used to induce remission. Patients who do not tolerate the initial course of maintenance pharmacotherapy should be treated with lithium. Valproate, quetiapine, or lamotrigine are recommended as maintenance therapy in individuals who do not respond to or tolerate lithium. Patients who fail to respond to multiple monotherapy agents can be treated with combination therapy of lithium or valproate and a second-generation antipsychotic. Most patients require maintenance treatment for many years, and some patients require it their entire lives. In addition, prevention of suicide is an important part of the treatment of bipolar disorder. All patients should be monitored for emergence or worsening of suicidal thoughts and behavior

A 79-year-old Army veteran with an extensive medical history presents to his primary care provider for cerumen irrigation and states "I am no value to anyone doc...why didn't they kill me in Korea?" For the past year, the patient reports an "unrelenting" loss of interest in living, reoccurring guilt regarding his military service, hopelessness, terminal insomnia, decreased appetite, decreased energy, decreased motivation, and low spirits. He denies contemplating suicide but states "but I am ready to go!" A comprehensive suicide assessment reveals that the patient is at moderate risk for suicide. Which of the following is the most likely diagnosis? A. Adjustment disorder B. Generalized anxiety disorder C. Major depressive disorder D. Persistent depressive disorder

C. Major depressive disorder Depression Screening Adults: All adults who have never been screened for depression should be screened (USPSTF) PHQ-9 is an assessment tool According to the Diagnostic and Statistical Manual of Mental Disorders, 5th Edition, a major depressive episode occurs when an individual experiences either depressed mood or anhedonia for two weeks or more while concurrently displaying at least five of the following symptoms: depressed mood, loss of pleasure (anhedonia), guilt or worthlessness, change in appetite, hypersomnia or insomnia, fatigue, change to the speed of bodily movement (psychomotor agitation/retardation), diminished concentration, and thoughts of suicide. Once diagnosed, combining treatment with psychotherapy and pharmacotherapy offers the best results. Selective serotonin reuptake inhibitors are the first-line treatment. However, selective serotonin reuptake inhibitors do not reach full effect until at least 4-6 weeks of administration. For severe depression, psychiatric procedures (such as electroconvulsive therapy, transcranial magnetic stimulation or ketamine administration) may be necessary. With appropriate treatment, roughly four out of five patients with major depressive disorder will experience improved mood, significant symptom reduction. However, approximately 50% of patients leave treatment prematurely.

A man presents to a counselor because he has difficulty completing tasks. He reports prioritizing perfection and orderliness at the expense of efficiency. He also tends to hoard money and is anxious about discarding things he no longer uses. He says that these difficulties are impacting his ability to work and his relationship with his wife. Which of the following is the most likely diagnosis? A. Generalized anxiety disorder B. Obsessive-compulsive disorder C. Obsessive-compulsive personality disorder D. Paranoid personality disorder

C. Obsessive-compulsive personality disorder The diagnostic criteria according to the Diagnostic and Statistical Manual, Fifth Edition (DSM-5) require at least four of the following: preoccupation with details, rules, lists, order, organization, or schedule to the point that the major point of the activity is lost; perfectionism that interferes with task completion; excessive devotion to work and productivity to the exclusion of leisure activities and friendships (not accounted for by obvious economic necessity); overly conscientious and inflexible about matters of mortality, ethics, or values (not accounted for by cultural or religious identification); unable to discard worn-out or worthless objects when they have no sentimental value; reluctance to delegate tasks or to work with others unless they submit to exactly his or her way of doing things; adoption of a miserly spending style toward self and others (money is viewed as something to be hoarded for future catastrophes); and showing rigidity and stubbornness. Some patients with obsessive-compulsive personality disorder have comorbid obsessive-compulsive disorder but most do not. First-line management of obsessive-compulsive personality disorder is psychotherapy (specifically cognitive-behavioral therapy). Pharmacologic therapy can be used to treat comorbid psychiatric conditions, such as unipolar depression and anxiety.

A 30-year-old man has a history of hypomanic episodes and major depressive episodes. He is diagnosed with bipolar disorder type 2. Which of the following is the most efficacious treatment for an acute hypomanic episode? A. Fluoxetine B. Lithium C. Olanzapine D. Valproate

C. Olanzapine Bipolar disorder type 2 is a mood disorder that is defined by at least one hypomanic episode and at least one major depressive episode in the absence of manic episodes. Patients with manic episodes have bipolar disorder type 1. Risk factors for bipolar disorder include family history, advancing paternal age (associated with increased genetic mutations), and stressful life events (childhood physical or sexual abuse). Hypomanic episodes are characterized by changes in mood, energy, activity, behavior, sleep, and cognition that are similar to those of mania but less severe. Furthermore, psychotic symptoms, such as hallucination and delusions, do not occur in hypomania, and hypomania never necessitates hospitalization. Patients with these features are classified as manic. Hypomania generally begins suddenly and progresses quickly over one to two days. Episodes typically resolve within several weeks.

What part of the history and physical is required for the diagnosis of antisocial personality disorder? A. Female sex B. Male sex C. Older than 18 years old D. Younger than 18 years old

C. Older than 18 years old Antisocial personality disorder is characterized by an adult, over the age of 18, who has complete disregard for the norms, values, and laws of society. Patients with antisocial personality disorder are most commonly men who typically have a history of conduct disorder in their teens. Risk factors for antisocial personality disorder include a family history of antisocial personality disorder, an alcoholic father, or adult criminal offenses in the family. The behaviors patients with antisocial personality disorder display may include impulsive behavior, spouse abuse, lack of remorse, frequent lying, and substance abuse. Treatment is focused on psychotherapy. There are no specific psychological tests that can be used to diagnose antisocial personality disorder, often because the patients do not have insight into their own behaviors. There are no definitive pharmacological therapies that are effective in treating antisocial personality disorder, however, certain antidepressants or antipsychotic agents have been used to reduce aggression. Less than half of patients show improvement in their antisocial behaviors by the time they reach their 40s, however, prognosis varies for each patient.

A 23-year-old woman presents to the emergency department with chest pain and palpitations that have resolved since her presentation. She was stuck in traffic when she noticed her heart racing accompanied by sweating, chest pain, and dizziness. She was extremely fearful because she has never felt like this before. There is no significant family or personal medical history. Physical examination, electrocardiogram, and laboratory studies are normal at this time. Which of the following is the most likely diagnosis? A. Agoraphobia B. Myocardial infarction C. Panic attack D. Panic disorder

C. Panic attack Patients with panic attack usually present with chest pain, dizziness, flushes, chills, feeling of choking, nausea, abdominal pain, numbness, tingling, palpitations, sweating, shortness of breath, and trembling. Physical examination may reveal signs of sympathetic state including hypotension, tachycardia, tachypnea, tremors, cool and clammy skin. Electrocardiogram may be normal or may show sinus tachycardia. Complete blood count, comprehensive metabolic panel, thyroid stimulating hormone, troponin, and creatinine kinase are usually within normal range. The Diagnostic and Statistical Manual of Mental Disorders, 5th edition criteria for diagnosing panic attacks include a sudden onset of fear or discomfort accompanied by four of the following symptoms: fear of dying, fear of losing control, derealization/depersonalization, chest pain, dizziness, a feeling of choking, flushes or chills, gastrointestinal discomfort, sweating, paresthesia, dyspnea, and tachycardia. Treatment is with antidepressants, including selective serotonin receptor inhibitors or serotonin-norepinephrine receptor inhibitors, and psychotherapy, including cognitive behavioral therapy and exposure therapy.

A 22-year-old woman presents to the clinic concerned about recent episodes of intense fear and discomfort. She has had several of these attacks that reach a peak intensity within minutes and last less than an hour. During these episodes, she experiences chest pain and palpitations. Although she reports some concern about having future panic attacks, the patient states she generally does not feel anxious. Which of the following is the most likely diagnosis? A. Generalized anxiety disorder B. Illness anxiety disorder C. Panic disorder D. Somatic symptom disorder

C. Panic disorder Patients with panic disorder may initially be treated with cognitive-behavioral therapy, antidepressant medication, or both. The decision to use cognitive-behavioral therapy or pharmacologic therapy is based on availability and patient preference. Individuals who do not respond to one modality should try the other modality. The recommended long-term first-line pharmacologic therapy for panic disorder is a selective serotonin reuptake inhibitor, such as sertraline or fluoxetine. The therapeutic effect of these medications typically takes two to four weeks, and clinical response can take as long as eight to 12 weeks for some patients. In addition, therapeutic response can continue to increase for six to 12 months. For patients who do not respond adequately after four to six weeks of a selective serotonin reuptake inhibitor at the maximally tolerated dose and still want to continue pharmacologic therapy, a different selective serotonin reuptake inhibitor or serotonin-norepinephrine reuptake inhibitor is suggested rather than other medications. Long-acting benzodiazepines are recommended for augmentation in patients with a partial response to a maximally tolerated dose of a selective serotonin reuptake inhibitor. However, patients with a history of abusing alcohol or other substances should not be treated with benzodiazepines due to their potential for dependence and tolerance. Adjunctive agents that can be used in patients with a history of substance use disorder include gabapentin, pregabalin, or mirtazapine. The treatment for acute panic attacks is benzodiazepines, such as lorazepam or alprazolam. Panic disorder is a recurrent or chronic disease in most cases. Predictors of remission include female sex, absence of ongoing life stressors, low initial frequency of attacks, and subthreshold panic.

A 32-year-old woman presents to the clinic with depressed mood for the past two and a half years. She states that she has been sleeping excessively, had poor appetite, and experienced a lack of concentration during this time. Which of the following is the most likely diagnosis? A. Bipolar disorder type 2 B. Cyclothymic disorder C. Persistent depressive disorder D. Unipolar depression

C. Persistent depressive disorder The diagnostic criteria for persistent depressive disorder according to the Diagnostic and Statistical Manual Fifth Edition (DSM-5) is at least two years of depressive symptoms without any symptom-free periods of more than two months. Patients must have depressed mood and at least two of the following additional depressive symptoms: poor concentration or indecisiveness, feelings of hopelessness, changes in appetite, sleep disturbances (insomnia or hypersomnia), fatigue, and low self-esteem. Therefore, this diagnosis includes individuals who have had unipolar major depression lasting at least two years. Patients with persistent depressive disorder do not have symptoms of mania, hypomania, or psychotic symptoms, such as hallucinations or delusions. The best treatment is combination therapy with psychotherapy and pharmacologic therapy. Selective serotonin reuptake inhibitors, such as sertraline or escitalopram, are first-line pharmacologic agents. Second-line agents include serotonin and norepinephrine reuptake inhibitors, bupropion, and tricyclic antidepressants. The best modalities of psychotherapy for persistent depressive disorder are behavioral therapy and interpersonal therapy. Persistent depressive disorder often follows a chronic course and is much less likely to resolve than major depressive disorder.

What is the initial treatment of choice for a mother with factitious disorder imposed on her child? A. Lorazepam B. Olanzapine C. Psychotherapy D. Sertraline

C. Psychotherapy Munchausen Syndrome by Proxy (Medical Child Abuse) Parent is causing or reporting the presenting nonorganic symptoms Morbidity and mortality can be high Symptoms disappear when the child is not with the caregiver creating the symptoms

A 30-year-old woman presents to the clinic stating she has been in a depressed mood for the past month. She states that other symptoms include inability to sleep, decreased interest in normally pleasurable activities, feeling guilty, lack of concentration, and increased appetite. She denies prior symptoms of hypomania or mania and denies suicidal ideation. Which of the following is the best treatment? A. Mirtazapine B. Psychotherapy C. Psychotherapy and sertraline D. Sertraline

C. Psychotherapy and sertraline Unipolar depression is defined by a depressed mood or anhedonia (loss of pleasure) with at least five associated depressive symptoms, including fatigue, sleep disturbances (hypersomnia or insomnia), significant weight changes, feelings of guilt, loss of interest in normally pleasurable activities, decreased concentration, psychomotor retardation (slowed speech or movement), and suicidal thoughts or ideation. The symptoms must be present most days for at least two weeks to diagnose major depressive disorder. Furthermore, symptoms must cause clinical distress or impairment in social or occupational function, must not be the result of another medical condition or substance, and must not be better explained by another psychiatric condition. The absence of mania or hypomania distinguishes unipolar depression from bipolar disorder.

A 36-year-old woman with a past psychiatric history of bipolar disorder (diagnosed at age 28) and schizophrenia (diagnosed at age 30) presents to the emergency department reporting "voices that laugh about my dad raping me...raping me... which makes me want to thrill myself." She is nonadherent to lurasidone and is admitted to inpatient psychiatry. During her initial psychiatric assessment, the patient notes depressed mood and decreased motivation to get out of bed for the past month. She also notes intense guilt, decreased appetite, sluggish movement, and inability to concentrate amid thoughts of suicide. On mental status exam, she appears disheveled and older than her stated age, has atonal speech (with frequent clangs), describes her mood as "depressed-a-mess," displays blunt affect, loose thought process, and momentarily responds to an absent "voice" during interview. What is the primary diagnosis for this patient? A. Major depressive episode with psychotic features B. Mixed episode of bipolar disorder C. Schizoaffective disorder D. Schizophrenia with comorbid major depression

C. Schizoaffective disorder To meet the Diagnostic and Statistical Manual of Mental Disorders, 5th Edition definition of schizoaffective disorder, a patient must simultaneously meet criteria for schizophrenia (two or more of the following: delusions, hallucinations, disorganized speech, disorganized or catatonic behavior, or negative symptoms) and an affective disorder (depression or mania) but must have periods of two weeks or more in which there is psychosis without depression or mania. However, mood symptoms must be present for the majority of the time in which schizophrenia is active and not in remission (so if the patient decompensates, then the mood symptoms should also arise). If the mood symptoms do not present with decompensation and instead emerge during a periods of time in which the psychosis is controlled of in remission, then that mood episode is evidence of a comorbid affective disorder, and not schizoaffective disorder. The initial treatment for schizoaffective disorder is antipsychotics (such as lurasidone). Antidepressants are introduced as a second-line treatment if the psychosis does not go into full remission or lingers. Full remission is possible with appropriate antipsychotic therapy, but becomes increasingly difficult with each subsequent episode of psychosis. As with schizophrenia, schizoaffective patients with an early age of onset or long periods of decompensated psychosis may become refractory to treatment. Refractory schizoaffective disorder is extremely debilitating and requires long-term involuntary hospitalization or commitment to mental health facility.

A 24-year-old woman presents to the clinic and is diagnosed with unipolar depression. She is treated with fluoxetine. Which of the following is a common adverse effect of this medication class? A. Hyperglycemia B. Seizures C. Sexual dysfunction D. Weight loss

C. Sexual dysfunction Studies have shown that about 50% of patients experience at least one adverse effect but only 10-20% discontinue the medication due to adverse effects. It is unclear whether selective serotonin reuptake inhibitors used in the treatment of depression raise or lower the risk of suicide. Selective serotonin reuptake inhibitors can also prolong the QT interval. Serotonin syndrome is a potentially lethal condition caused by overstimulation of central and peripheral serotonin receptors. It is usually caused by multiple medications that increase serotonergic transmission, rather than solely by selective serotonin reuptake inhibitor monotherapy. The syndrome often occurs after initiating a new medication or increasing the dose of a current medication. Clinical findings of serotonin syndrome may include anxiety, agitation, delirium, diaphoresis, tachycardia, hypertension, hyperthermia, gastrointestinal symptoms, tremor, muscle rigidity, myoclonus, and hyperreflexia. Selective serotonin reuptake inhibitors are usually dosed once daily and are often taken in the morning to avoid insomnia. Patients must take selective serotonin reuptake inhibitors daily rather than on an as-needed basis to increase the chance of efficacy. Patients should also be educated that although some response to therapy may be evident within two weeks, it may take up to six weeks to see the full clinical benefit. Patients should be informed that abruptly discontinuing selective serotonin reuptake inhibitors can lead to dysphoria, dizziness, gastrointestinal symptoms, fatigue, and myalgias. Selective serotonin reuptake inhibitors should be tapered over two to four weeks prior to discontinuation. As an example, a patient taking 60 milligrams per day may take 40 milligrams per day for one week, then 20 milligrams per day for a week, and then discontinue therapy

A 30-year-old man presents to the clinic for treatment of unipolar major depression. It is decided that a serotonin-norepinephrine reuptake inhibitor will be the initial treatment. What is an appropriate trial period prior to assessing the efficacy of an antidepressant? A. Four weeks B. One week C. Six weeks D. Three weeks

C. Six weeks Antidepressants, including serotonin-norepinephrine reuptake inhibitors, should generally be used for six weeks prior to switching therapy (if there is lack of efficacy). Clinicians should educate patients to not abruptly discontinue antidepressants due to the possible adverse effects, including chills, dizziness, dysphoria, fatigue, gastrointestinal distress, and myalgias. There are no specific medical or laboratory tests that are required before starting serotonin-norepinephrine reuptake inhibitors. Drug serum concentrations are not routinely monitored. Adverse effects of serotonin-norepinephrine reuptake inhibitors include nausea, an increase in blood pressure, dizziness, diaphoresis, and sexual dysfunction. Nausea is the most common adverse effect and may be decreased by administering the serotonin-norepinephrine reuptake inhibitors with food. Nausea appears to diminish over time. Blood pressure should be assessed prior to starting and throughout treatment. Blood pressure should be checked every one or two weeks for the first month and then every one or two months for the next six months. Duloxetine does not have an impact on blood pressure. Sexual dysfunction may include decreased libido, delayed ejaculation in men, anorgasmia in women, and decreased arousal. The association of antidepressant medications with suicidality is age-specific. Among children, adolescents, and young adults, there is evidence that the onset of suicidality is increased during the first few weeks of antidepressant treatment. There is no detectable effect in middle aged adults, and there is a protective effect in older adults. However, in younger patients, there is also clear risk of suicidality in untreated depressed individuals. All patients initiating antidepressants should receive follow-up within two to four weeks. Serotonin-norepinephrine reuptake inhibitors should be started at a low dose and slowly titrated upwards to the lowest effective dose. Patients with recurrent depression who recover from an episode of major depression should generally receive maintenance treatment with the full dose that resolved the episode

A 45-year-old man is brought to the emergency department because of chest pain for the past 3 hours. The patient has been seen several times in the last year for similar symptoms, however, each time cardiac evaluation is normal. History reveals no cardiac risk factors. The patient says he barely leaves his house and had to quit his job because he fears his symptoms of chest pain will begin and he will not be able to reach a hospital. After thorough evaluation, all tests are negative. The patient becomes frustrated and storms out of the hospital demanding a second opinion. Which of the following is the most likely diagnosis? A. Adjustment disorder B. Conversion disorder C. Somatic symptom disorder D. Panic disorder E. Post-traumatic stress syndrome

C. Somatic symptom disorder Somatic symptom disorder is a DSM-V diagnosis characterized by excessive preoccupation with minor bodily symptoms. Patients often self diagnose themselves and do not believe the physician's negative work-up. Preoccupation with or fear of their symptomatology can often inhibit a patients ability to perform important activities, such as work, school activities, or family and social responsibilities. The preoccupation must be present for at least six months. Treatment consists of regular follow-up visits and offering psychotherapy. The patient should not be told that these symptoms are imaginary. This diagnosis was previously known as hypochondria, but in the DSM-V that diagnosis is not possible. A similar DSM-V diagnosis to somatic symptom disorder is illness anxiety disorder which is characterized by intense anxiety about an undiagnosed condition.

_______________ pharmacologic therapy may result in agranulocytosis (severe leukopenia) and high risk for infection. This medication is the second leading cause of agranulocytosis by a psychotropic medication, and strict guidelines DO NOT exist as they do for clozapine.

Carbamazepine Check CBC regularly carbamazepine is the second leading cause of agranulocytosis by a psychotropic medication, yet strict guidelines do not exist for monitoring as they do for clozapine. Initially, the manufacturer of carbamazepine had extensive guidelines for monitoring for blood dyscrasias.

A 45-year-old man comes to the psychiatric clinic because of depressed mood. He is diagnosed with depression and begins treatment with fluoxetine. After 3 months of therapy, he comes to the clinic because he is worried about his sexual performance, which has decreased considerably since he began taking this drug. At his follow-up appointment, it is decided that he is to switch to a different drug called trazodone. Which of the following receptors is responsible for the decreased sexual activity experience by the patient while taking fluoxetine? A. 5-HT3 B. 5-HT1b C. 5-HT2c D. 5-HT2a E. 5-HT1d

D. 5-HT2a Anorgasmia is one of the main adverse effects experienced by patients taking selective serotonin reuptake inhibitors, and it's thought to be caused by stimulation of 5-HT2A receptors.

A 61-year-old patient with a history of alcohol use disorder is also diagnosed with Wernicke-Korsakoff syndrome. This syndrome is a brain disorder due to thiamine (vitamin B1) deficiency composed of two different conditions. When symptoms of Wernicke encephalopathy are left untreated, the symptoms may progress to Korsakoff syndrome. Which of the following findings is most commonly associated with Korsakoff syndrome? A. Alcohol withdrawal symptoms B. Confusion and attention defecit C. Demyelination causing gait ataxia D. Hallucinations and inability to form new memories E. Paresthesias in the lower extermities

D. Hallucinations and inability to form new memories Korsakoff syndrome, which develops as Wernicke encephalopathy symptoms disappear, is most likely to be associated with hallucinations and inability to form new memories. Also known as Korsakoff psychosis, it results from permanent damage to areas of the brain involved with memory. The mammillary bodies and medial dorsal nucleus of the thalamus are commonly affected. Wernicke-Korsakoff syndrome can occur from chronic alcohol use, causing a deficiency in thiamine. Wernicke encephalopathy is characterized by confusion, gait ataxia, and ophthalmoplegia. If left untreated, Korsakoff syndrome is characterized by amnesia, personality change, and confabulation. Mnemonic for Wernicke-Korsakoff syndrome: Wernicke wears a reversibleCOAT and Korsakoff covers with a black KAP. Reversible COAT= reversible if treated, Confusion, Ophthalmoplegia, and ATaxia. blacK KAP= "Konfabulation", Amnesia, and Personality changes.

A 34-year-old G1P1 has been diagnosed with postpartum depression. She was prescribed sertraline, a selective serotonin reuptake inhibitor (SSRI). Although SSRIs are the first line agents to treat this disorder, which of the following side effects may the patient experience? A. Sleep disturbances, increased blood pressure, constipation, and abnormal vision B. Sedation, weight gain, dry mouth, convulsion, coma, cardiotoxicity, constipation, and sexual dysfunction C. CNS stimulation and hypertensive crisis D. Insomnia, jitteriness, nausea, appetite suppression, sexual dysfunction, and headaches

D. Insomnia, jitteriness, nausea, appetite suppression, sexual dysfunction, and headaches Insomnia, jitteriness, nausea, appetite suppression, sexual dysfunction, and headaches are the most common adverse effects of SSRIs which include paroxetine, citalopram, escitalopram, sertraline, and fluoxetine Major Takeaway: Insomnia, jitteriness, nausea, appetite suppression, sexual dysfunction, and headaches are some adverse effects of SSRIs. Serotonin syndrome is less common, but important to remember. A useful mnemonic to remember for some common ones is: S - Serotonin syndrome S - Stimulate CNS (jitteriness etc.) R - Reproductive (sexual) dysfunction I - Insomnia

A 46-year-old man presents to his primary care clinic with concerns about insomnia for two months. While interviewing the patient, he also mentions that he has not been eating as much lately, feels tired all the time, has trouble focusing at work, and does not feel like meeting up with his friends or family anymore. He reports these symptoms started around the same time as his insomnia. His physical exam is unremarkable, and vital signs are all within normal limits. Which of the following is the most likely diagnosis? A. Bipolar II disorder B. Cyclothymia C. Dysthymia D. Major depressive disorder

D. Major depressive disorder This patient has major depressive disorder, which is the most common of the depressive disorders. There needs to be at least a two-week history of five out of nine depressive symptoms that impede a patient's ability to function to diagnose a patient with major depressive disorder. These symptoms include a sad mood, lack of interest in activities (anhedonia), changes in sleeping patterns, lack of energy, changes in appetite, guilt, difficulty concentrating, psychomotor agitation, and suicidal ideation or self-harm. There are no physical exam findings or laboratory tests associated with major depressive disorder. The patient health questionnaire (PHQ-9) is a screening tool used to diagnose major depressive disorder and to monitor progress during treatment. Treatment includes psychotherapy therapy and antidepressant medications. Selective serotonin receptor inhibitors are the first-line medications for major depressive disorder. Common side effects of this class as a whole include gastrointestinal upset, sexual dysfunction, and weight gain.

A 26-year-old man with no known medical history is brought to the emergency department after being resuscitated from opioid overdose and is admitted to inpatient psychiatry for stabilization during acute opioid withdrawal. After seven days, the patient ceases to show signs of opioid withdrawal and requests discharge. During disposition planning, the patient states "I don't ever want to go through withdrawal again!" Which treatment can be used for long-term prevention of opioid relapse without risk of withdrawal if discontinued? A. Buprenorphine/naloxone B. Methadone C. Naloxone D. Naltrexone

D. Naltrexone Opioid Withdrawal Flu-like illness Abdominal cramps Diarrhea Mydriasis Piloerection Yawning Rx: methadone, buprenorphine, clonidine for iatrogenic withdrawal, other nonopioid adjuncts Naltrexone is a mu-receptor antagonist that blocks the effect of opioids, decreases cravings, and is available in long-acting injectable form, thus making it ideal for preventing opioid relapse. Uniquely, naltrexone is metabolized to 6-beta-naltrexol, which is an agonist of opioid receptors, and thus prevents cravings for opioids as well. However, naltrexone can lead to opioid withdrawal during introduction. Therefore, a trial of low-dose oral naltrexone should be introduced after withdrawal has ended but before beginning intramuscular naltrexone. The most salient sign of naltrexone-induced opioid withdrawal is acute diarrhea, which some patients find intolerable, and may discourage future cooperation with substance abuse treatment. Unlike methadone, naltrexone will not lead to withdrawal if discontinued. Naltrexone has also been shown to decrease the pleasure of ethanol and thus is also indicated in the treatment of alcohol use disorder.

A 42-year-old man reports to the behavioral health clinic for troubling side effects. He reports he has had a dry mouth and constipation since initiating his medication. Additionally, monitoring over the past six months has shown a weight gain of 12 pounds with increases in hemoglobin A1C to 6.2% and in low-density lipoprotein to 145 mg/dL. Which of the following medications is most likely causing this patient's symptoms? A. Aripiprazole B. Haloperidol C. Loxapine D. Olanzapine

D. Olanzapine Antipsychotics, which are used to treat a loss of contact with reality (i.e., psychosis), work by blocking the postsynaptic dopamine D2 receptors in the brain. Delusions, hallucinations, and disorganized thoughts predominate the symptoms of psychosis. In addition to dopamine receptors, antipsychotics affect serotonin, alpha-1, histamine, and muscarinic receptors. Side effects are proportionally related to the degree by which these receptors are affected. Typical antipsychotics (haloperidol, chlorpromazine) are more likely to cause side effects related to dopamine blockade in the nigrostriatal tract and manifest as extrapyramidal symptoms. These symptoms include pseudoparkinsonism (bradykinesia, rigidity, tremor), acute dystonia, and akathisia (sense of restlessness). This class, also called first-generation antipsychotics, is more likely to cause life-threatening neuroleptic malignant syndrome and tardive dyskinesia. These conditions, however, can also occur with second-generation (atypical) antipsychotics. Risperidone, aripiprazole, asenapine, lurasidone, and paliperidone are atypical antipsychotics that may carry a higher risk of tardive dyskinesia than others in this category. These same medications are more likely to cause extrapyramidal syndrome than other second-generation antipsychotics, with risperidone carrying the highest risk. Metabolic changes (i.e., weight gain, diabetes, and dyslipidemia) are commonly associated with second-generation antipsychotics. Both an increased appetite and altered metabolic control have been discovered secondary to use of this class. Patients treated with clozapine and olanzapine are at highest risk of developing adverse metabolic effects, whereas aripiprazole, lurasidone, pimavanserin, and ziprasidone are the lowest risk medications in this regard. Anticholinergic effects related to second-generation antipsychotics include dry mouth, constipation, blurred vision, and urinary retention. These adverse effects occur secondary to muscarinic receptor antagonism, which is most frequent and severe with clozapine. Olanzapine and quetiapine also pose a high risk for this side effect.

A 20-year-old man presents to the psychiatric emergency center with hallucinations and delusions. On exam, he demonstrates blunted affect, apathy, and circumstantiality. Chart review shows that he has had symptoms for at least six months. Toxicology screen is negative. He is diagnosed with schizophrenia. Which of the following is the best treatment for the management of this acute psychotic episode? A. Clozapine B. Fluoxetine C. Lorazepam D. Olanzapine

D. Olanzapine Schizophrenia Two or more symptomsDelusions, hallucinations, disorganized speech, grossly disorganized or catatonic behavior, negative symptoms Brief psychotic disorder: < 1 month Schizophrenia: > 6 months Schizophreniform disorder: 1-6 months Schizoaffective disorder: psychosis + mania or depression

A 19-year-old woman presents to your clinic with a concern about her weight. After discussion, she reports eating large quantities of food and then forcing herself to vomit about one to two times per week. She says that this has been going on for six months and is increasing in frequency. She meets the criteria for a diagnosis of bulimia nervosa. Which of the following physical exam findings is most likely to be seen on exam? A. Bradycardia B. Lanugo C. Low body mass index D. Parotid gland enlargement

D. Parotid gland enlargement Bulimia Nervosa Patient with a history of binge eating followed by purging via forced vomiting, purging via laxative misuse, driven exercise, or fasting PE will show dental erosions and callused knuckles Diagnosis requires at least one episode of binge eating with inappropriate compensatory behavior per week for a minimum of 3 months in a patient whose self-evaluation is unduly influenced by body shape or weight Treatment is cognitive behavioral therapy, SSRIs, or both

A 25-year-old man presents with depressive symptoms after witnessing the death of his brother in a motor vehicle collision four months ago. The patient reports two months of repetitive distressing dreams about the event and frequent dissociative reactions when he is a passenger in a vehicle. Which of the following is the most likely diagnosis? A. Acute stress disorder B. Generalized anxiety disorder C. Obsessive-compulsive disorder D. Post-traumatic stress disorder

D. Post-traumatic stress disorder Post-Traumatic Stress Disorder (PTSD) Sx duration > 1 month Persistently reexperiencing of event Persistently ↑ arousal Avoidance of stimuli ↑ risk for suicide, substance use Trauma-focused psychotherapy (e.g., exposure therapy, cognitive processing) is first-line treatment in newly diagnosed patients. Pharmacologic therapy includes the use of selective serotonin reuptake inhibitors (SSRIs) (e.g., fluoxetine, sertraline) or serotonin-norepinephrine reuptake inhibitors (e.g., duloxetine, desvenlafaxine). Insomnia may be treated with prazosin in addition to other medical therapies or alone. If pharmacologic intervention effectively controls symptoms, management should be continued for at least six months to one year to prevent relapse or recurrence. If an SSRI is ineffective, another medication in this class should be tried prior to choosing an alternative agent. If a patient does not have success with monotherapy, a second-generation antipsychotic (e.g., quetiapine, risperidone) may be added to augment medication efficacy.

A 6-year-old boy is brought to the emergency department by his mother who is concerned that "he has blood in his urine." In the past three months, he has had multiple visits to the ED for the same complaint with extensive evaluations resulting in completely negative workups for hematuria or other abnormalities. It is determined that she has been contaminating the urine samples with her own blood. Which of the following is the most appropriate next step in management? A. Family support therapy B. Legal intervention C. Repair of iatrogenic harm D. Report to child protection services

D. Report to child protection services Munchausen Syndrome by Proxy (Medical Child Abuse) Parent is causing or reporting the presenting nonorganic symptoms Morbidity and mortality can be high Symptoms disappear when the child is not with the caregiver creating the symptoms

A 22-year-old man presents to a primary care clinician and states he is concerned he will develop lung cancer. He explains that for the past seven months he has spent numerous hours researching lung cancer online and has been to several doctors. He has had several computed tomography scans of his chest done that have not identified any pathology. He denies any symptoms and denies any history of smoking cigarettes. Which of the following is part of the recommended treatment for the most likely diagnosis? A. Beginning antipsychotic pharmacologic therapy B. Ordering frequent diagnostic testing to reassure the patient C. Referring to several specialists to reassure the patient D. Scheduling regular appointments with one health care provider

D. Scheduling regular appointments with one health care provider Illness Anxiety Disorder Hypochondriasis Preoccupied with serious illness, despite negative exam or testing Evaluate for other medical diagnosis Management is CBT, psychotherapy, antidepressants

What is the only anti-depressant used for dermatotillomania?

Fluoxetine

A 20-year-old man presents to the clinic accompanied by family members who are concerned about his lifestyle. His family states that he has no friends and does not ever go out. The patient has a flat affect during the encounter and states that he does not desire relationships. Which of the following is the most likely diagnosis? A. Avoidant personality disorder B. Major depressive disorder C. Paranoid personality disorder D. Schizoid personality disorder

D. Schizoid personality disorder Schizoid Personality Disorder Detachment from social relationships Restricted range of expression Solitary activities Emotionally cold or flat Avoidance of close relationships Pleasure in few activities Psychotherapy is first-line treatment for schizoid personality disorder. However, due to poor insight, individuals with schizoid personality disorder rarely seek treatment.

A 24-year-old man is diagnosed with post-traumatic stress disorder after experiencing a motor vehicle collision. He endorses flashbacks, insomnia, and an intense fear of getting in another vehicle. He avoids being in motor vehicles and, as a result, has missed work for six weeks since the incident. Which of the following medications is most appropriate in treating this patient? A. Aripiprazole B. Clomipramine C. Clozapine D. Sertraline

D. Sertraline First-line treatment in newly diagnosed patients involves either trauma-focused psychotherapy or medication management using selective serotonin reuptake inhibitors (e.g., fluoxetine, sertraline) or serotonin-norepinephrine reuptake inhibitors (e.g., duloxetine, desvenlafaxine). Trauma-focused therapies involve exposure therapy, cognitive processing, and eye movement desensitization and reprocessing. If a predominant symptom is insomnia, patients may be treated with prazosin, either with a selective serotonin reuptake inhibitor or alone. Patients whose symptoms are effectively controlled with medication management should continue this treatment for at least six months to one year to prevent relapse or recurrence. Failure of one medication in the selective serotonin reuptake inhibitor class does not preclude treatment with another, and an alternative may be started. If a patient does not have success with monotherapy, a second-generation antipsychotic (e.g., quetiapine, risperidone) may be added to augment medication efficacy.

A 25-year-old woman presents to the clinic with recurrent episodes of abrupt and intense fear. She says the episodes last about 10 minutes each time. She states that she does not want to go into public places because she fears she will have one of these episodes. She has had extensive medical evaluation without an identifiable etiology and reports no substance use. Which of the following is considered first-line pharmacologic therapy? A. Gabapentin B. Lorazepam C. Mirtazapine D. Sertraline

D. Sertraline The initial treatment of panic disorder typically includes cognitive-behavioral therapy, antidepressant medication, or both. The decision to use cognitive-behavioral therapy or pharmacologic therapy is based on availability and patient preference. Individuals who do not respond to one modality should try the other modality. The recommended long-term first-line pharmacologic therapy for panic disorder is a selective serotonin reuptake inhibitor, such as sertraline or fluoxetine. For patients who do not respond adequately after four to six weeks of a selective serotonin reuptake inhibitor at the maximally tolerated dose and still want to continue pharmacologic, a different selective serotonin reuptake inhibitor or serotonin-norepinephrine reuptake inhibitor is suggested rather than other medications. Long-acting benzodiazepines are recommended for augmentation in patients with a partial response to a maximally tolerated dose of a selective serotonin reuptake inhibitor. However, patients with a history of excessively using alcohol or other substances should not be treated with benzodiazepines due to their potential for dependence and tolerance. Alternative adjunctive agents include gabapentin, pregabalin, or mirtazapine. The treatment for acute panic attacks is benzodiazepines, such as lorazepam or alprazolam. Panic disorder is a recurrent or chronic disease in the majority of cases. Predictors of remission include female sex, absence of ongoing life stressors, low initial frequency of attacks, and subthreshold panic.

An 18-year-old woman with a history of a restrictive eating disorder is hospitalized due to recent, rapid weight loss. Her vital signs are normal on admission. A panel of diagnostic laboratory testing is sent to manage her condition as she begins a monitored nutrition rehabilitation program. Which of the following studies is most important to monitor for complications? A. Electrocardiogram B. Liver function tests C. Serum creatinine D. Serum phosphate

D. Serum phosphate While weight gain is the most important treatment for patients with anorexia nervosa, patients with severe weight loss must be closely monitored for refeeding syndrome in the first one to two weeks of nutritional rehabilitation and weight gain. Hallmarks of refeeding syndrome include hypophosphatemia, hypokalemia, congestive heart failure, hemolysis, peripheral edema, rhabdomyolysis, and seizures. The primary cause of refeeding syndrome is hypophosphatemia, thus serum phosphate should be monitored closely. Patients with anorexia nervosa are already phosphate-depleted due to starvation, and consumption of carbohydrates during nutritional rehabilitation can lead to a further decrease of serum phosphorus as insulin triggers cellular uptake of phosphate. Potentially fatal consequences of hypophosphatemia include tissue hemolysis, tissue hypoxia, myocardial dysfunction, respiratory failure and seizures, with cardiac complications being the most common fatal sequelae. Patients with evidence of refeeding syndrome should have their nutritional support reduced while abnormalities like hypophosphatemia, hypokalemia, and hypomagnesemia are corrected. Risk for refeeding syndrome gradually reduces after consistent nutritional intake and weight gain. For long-term success in treating anorexia, psychotherapy and pharmacotherapy are also standard approaches to managing these patients.

A 24-year-old woman presents to her primary care provider complaining of epigastric pain and nausea for the past several years. She has seen several gastroenterologists who have obtained advanced imaging and performed numerous endoscopies without identifying the cause of her symptoms. She has tried pharmacologic therapies, which help some. She states that she is very concerned about these symptoms and frustrated with the health care providers she has seen. She says that she loses sleep and has been unable to function at work because she spends most of her day reading about her symptoms. Which of the following is the most likely diagnosis? A. Conversion disorder B. Illness anxiety disorder C. Obsessive-compulsive disorder D. Somatic symptom disorder

D. Somatic symptom disorder Somatic Symptom Disorder (formerly Somatoform Disorder) Genuine sx with no identifiable cause Precipitating emotional event often precedes somatic symptoms F > M Management involves a combination of behavior modification, and psychological and social therapy

A 55-year-old man presents to the clinic with his daughter who states that her father was in a traumatic car collision several years ago. Since that time, he refuses to ride in cars or drive due to the fear of getting in another collision. The patient's daughter explains that he experiences significant distress and panic during car rides and avoids discussing it. She is concerned that her father's well-being is significantly impacted, as he almost never leaves the house. Which of the following is the most likely diagnosis? A. Agoraphobia B. Panic disorder C. Post-traumatic stress disorder D. Specific phobia

D. Specific phobia Specific phobia is an anxiety disorder where patients associate significant fear with a particular object or situation and experience symptoms of panic during exposure to the stimulus, anticipation of the stimulus, or even hearing the stimulus spoken of. This association invariably leads to avoidance behavior. The Diagnostic and Statistical Manual of Mental Disorders, 5th Edition delineates five primary specifiers, or types, of phobias: animal (spiders, dogs, insects), natural environment (heights, storms, water), blood-injection-injury (needles, invasive procedures), situational (flying in airplanes, driving, enclosed spaces), and other (situations that could lead to choking or vomiting, loud sounds). Patients can have different focuses of their fear for the same stimulus, including disgust, danger or harm, or the experience of physical symptoms in the phobic situation.

A 19-year-old woman presents to your office for follow-up after an inpatient psychiatric hospitalization. While in the hospital, she was diagnosed with bipolar disorder and was started on lithium. She now has questions about side effects and monitoring. Which of the following should be monitored regularly while taking lithium? A. Hemoglobin and hematocrit B. Liver enzymes C. Potassium D. Thyroid function

D. Thyroid function Lithium FDA-approved indications Bipolar disorder, acute mania Bipolar disorder, maintenance treatment Off-label uses Bipolar, depression Major depressive disorder, adjunct Suicidality in mood disorders Typical therapeutic levels Mania: 1.0-1.5 mEq/L Depression: 0.6-1.0 mEq/L Maintenance: 0.7-1.0 mEq/L Monitoring Lithium levels Creatinine, urine specific gravity Thyroid function tests ECG in those age > 50 years

A 27-year-old woman presents to the clinic complaining of a depressed mood for the past two weeks. She states she has had significant weight gain, has been sleeping more than usual, has decreased interest in normally pleasurable activities, feels guilty, and has had decreased concentration. She states she had a hypomanic episode in the past while she was using amphetamines. Which of the following is the most likely diagnosis? A. Bipolar disorder type 1 B. Bipolar disorder type 2 C. Persistent depressive disorder D. Unipolar depression

D. Unipolar depression Unipolar depression is defined by a depressed mood or anhedonia (loss of pleasure) with at least five associated depressive symptoms, including fatigue, sleep disturbances (hypersomnia or insomnia), significant weight changes, feelings of guilt, loss of interest in normally pleasurable activities, decreased concentration, psychomotor retardation (slowed speech or movement), and suicidal thoughts or ideation. The symptoms must be present most days for at least two weeks to diagnose major depressive disorder. Furthermore, symptoms must cause clinical distress or impairment in social or occupational function, must not be the result of another medical condition or substance, and must not be better explained by another psychiatric condition. The absence of mania or hypomania distinguishes unipolar depression from bipolar disorder. However, if previous manic or hypomanic episodes were substance-induced, then those episodes do not meet the criteria for the diagnosis of bipolar disorder. Risk factors for major depressive disorder include family history and female sex. The highest incidence of major depression occurs in individuals between 20-40 years of age. The pathophysiology consists of alteration in neurotransmitters, such as serotonin, epinephrine, norepinephrine, and acetylcholine. There is a genetic component, as family history is a risk factor. Patients with unipolar depression typically suffer comorbid psychiatric disorders, such as anxiety disorders, post-traumatic stress disorder, obsessive-compulsive disorder, and substance use disorders. Pharmacologic therapy plus psychotherapy is recommended as the initial treatment of unipolar major depression. However, pharmacotherapy alone and psychotherapy alone are reasonable alternatives. There are several classes of antidepressant medications available, and each class generally has similar efficacy. Selective serotonin reuptake inhibitors, such as fluoxetine, escitalopram, and sertraline, are generally prescribed first due to their tolerability in studies. Selective serotonin reuptake inhibitors should be attempted for four to six weeks before the efficacy of the treatment is evaluated. Other classes of antidepressants include serotonin norepinephrine reuptake inhibitors, tricyclic antidepressants, and monoamine oxidase inhibitors. Electroconvulsive therapy can be used in patients who fail to respond to medical therapy, who have responded well to electroconvulsive therapy in the past, or who have severe symptoms. Electroconvulsive therapy is performed under general anesthesia and intentionally triggers a brief seizure. It is safe in pregnancy and in the elderly.

Which of the following is first-line treatment for a patient with acute severe mania? A. Electroconvulsive therapy B. Lamotrigine C.Lithium D. Valproate and olanzapine

D. Valproate and olanzapine Bipolar Disorder Lifelong, recurrent mood episodes of either mood pole Mania, hypomania Inflated self-esteem Decreased need for sleep Pressured speech Flight of ideas Excessive pleasurable activity

What infection may cause an acute onset of obsessive-compulsive symptoms?

Group A streptococcal infection, which may cause pediatric autoimmune neuropsychiatric disorder associated with streptococcal infections.

A 35-year-old man comes to the family medicine clinic because of compulsive actions for the past 6 months. He reports that he cannot stop thinking about stepping on pieces of chewing gum while walking alongside the street. He states that he often has to constantly check and re-check his car doors to make sure they are locked. He acknowledges that these actions are ridiculous. Which of the following is the most appropriate next step in management? A. Electroconvulsive therapy B. Haloperidol C. Benzodiazepine D. Buspirone E. Citalopram

E. Citalopram Obsessive compulsive disorder (OCD) is characterized by intrusive recurrent thoughts (obsessions) and impulsive behaviors (compulsions). First line pharmacological treatment for OCD is with selective serotonin reuptake inhibitors such as citalopram.

A 40-year-old woman is in the hospital recovering from a cholecystectomy performed 3 days ago that was complicated by cholangitis. She is being treated with IV piperacillin-tazobactam. She calls the nurse to her room because she says that her heart is racing. She also demands that someone come in to clean the pile of garbage off of the floor because it is attracting flies. The patient appears sweaty, distressed, and unable to remain still. She is oriented to person, but not place or time. Palpation of the abdomen shows no tenderness, rebound, or guarding. Her temperature is 38°C (100.4°F), pulse is 112/minute, respiration is 20/minute, and blood pressure is 150/90 mm Hg. Which of the following is the most likely diagnosis? A. Thyroid storm B. Hepatic encephalopathy C. Acute cholangitis D. Alcoholic hallucinosis E. Delirium tremens

E. Delirium tremens Delirium tremens begins 2 to 4 days into alcohol withdrawal and is a life-threatening condition treated with IV benzodiazepines. Like alcoholic hallucinosis, DT is characterized by hallucinations, but they occur with disorientation and abnormal vital signs.

A 65-year-old man is brought to the emergency department by ambulance because of chest pain and difficulty breathing. He was hit by a car after stumbling into the street at a busy intersection and sustained trauma to his chest. Medical history is significant for multiple hospitalizations for alcohol-related injuries and seizures. The patient is intubated in the emergency department for a flail chest segment and spends one week sedated in the intensive care unit. After a successful extubation, he is transferred to the floor where it is noted that he is confused, unsteady, and easily agitated. Physical examination shows asymmetric eye movements, most notably a lateral rectus palsy. The patient is oriented only to himself and is unable to walk without assistance. Which of the following is the most likely diagnosis? A. Delirium B. Delirium tremens C. Korsakoff psychosis D. Traumatic brain injury E. Wernicke encephalopathy

E. Wernicke encephalopathy Wernicke encephalopathy presents most commonly as ataxia, encephalopathy, and ophthalmoplegia (lateral rectus or conjugate gaze palsy), as this patient is showing signs of. If not treated, it can progress to anterograde/retrograde amnesia, nystagmus, and confabulations known as Korsakoff psychosis

T or F: Conversion disorder signs and symptoms are consistent with a neurologic disease and produces voluntary symptoms.

FALSE Coversion disorder signs and symptoms are NOT consistent with neurlogic disease Symptoms are involuntary No secondary gain

With valproate, it is important to monitor (CMP/LFT's) ________ regularly.

LFT's Can result in elevated LFTs and heaptitis

___________ -> can cause Ebstein's anomaly in pregnancy (cardiac abnormality)

Lithium

Lamotrigine is associated with _______________, which is a severe mucocutaneous rxn resulting in extensive necrosis and detachment of epidermis.

Steven's Johnson Syndrome Monitor for rash

__________ is a common adverse effect of lithium that is often associated with peak levels of lithium. Dividing the dose of lithium and decreasing caffeine can decrease this ADR.

Tremor

A previously healthy 22-year-old man presents to your office requesting treatment for attention deficit hyperactivity disorder. He says that he was diagnosed as a child and has tried to remain off medication, but he has a new job and is struggling to concentrate and complete tasks. Which of the following is the most appropriate initial therapy? A. Bupropion B. CBT C. Dextroamphetamine D. Nortriptyline

c. Dextroamphetamine Attention-Deficit/Hyperactivity Disorder (ADHD) Most common childhood behavioral disorder Inattentive Sxs, impulsive or hyperactive Sxs, or both Dx criteria Sxs present in 2 areas of interaction Sxs must present before 12 years Sxs > 6 months Sxs maladaptive or inappropriate for child's developmental stage Rx: medication (most effective), behavioral modification, environmental intervention


संबंधित स्टडी सेट्स

How to Get a 5 on the APUSH Exam

View Set

chapter 12 and 13 financial accounting

View Set

Principles of Management Chapter 13

View Set

Unit 13 Flocabulary: Yes We Can (Sentences Level 6)

View Set